Programs & Examples On #Maxrequestlength

Maximum value of maxRequestLength?

These two settings worked for me to upload 1GB mp4 videos.

<system.web>
    <httpRuntime maxRequestLength="2097152" requestLengthDiskThreshold="2097152" executionTimeout="240"/>
</system.web>
<system.webServer>
    <security>
        <requestFiltering>
            <requestLimits maxAllowedContentLength="2147483648" />
        </requestFiltering>
    </security>
</system.webServer>

javascript code to check special characters

Directly from the w3schools website:

   var str = "The best things in life are free";
   var patt = new RegExp("e");
   var res = patt.test(str);

To combine their example with a regular expression, you could do the following:

function checkUserName() {
    var username = document.getElementsByName("username").value;
    var pattern = new RegExp(/[~`!#$%\^&*+=\-\[\]\\';,/{}|\\":<>\?]/); //unacceptable chars
    if (pattern.test(username)) {
        alert("Please only use standard alphanumerics");
        return false;
    }
    return true; //good user input
}

How can I use console logging in Internet Explorer?

Since version 8, Internet Explorer has its own console, like other browsers. However, if the console is not enabled, the console object does not exist and a call to console.log will throw an error.

Another option is to use log4javascript (full disclosure: written by me), which has its own logging console that works in all mainstream browsers, including IE >= 5, plus a wrapper for the browser's own console that avoids the issue of an undefined console.

Run a .bat file using python code

This has already been answered in detail on SO. Check out this thread, It should answer all your questions: Executing a subprocess fails

I've tried it myself with this code:

batchtest.py

from subprocess import Popen
p = Popen("batch.bat", cwd=r"C:\Path\to\batchfolder")
stdout, stderr = p.communicate()

batch.bat

echo Hello World!
pause

I've got the batchtest.py example from the aforementioned thread.

What are good ways to prevent SQL injection?

My answer is quite easy:

Use Entity Framework for communication between C# and your SQL database. That will make parameterized SQL strings that isn't vulnerable to SQL injection.

As a bonus, it's very easy to work with as well.

Add / Change parameter of URL and redirect to the new URL

Some simple ideas to get you going:

In PHP you can do it like this:

if (!array_key_exists(explode('=', explode('&', $_GET))) {
  /* add the view-all bit here */
}

In javascript:

if(!location.search.match(/view\-all=/)) {
  location.href = location.href + '&view-all=Yes';
}

Measuring text height to be drawn on Canvas ( Android )

You must use Rect.width() and Rect.Height() which returned from getTextBounds() instead. That works for me.

How do I disable form resizing for users?

Use the FormBorderStyle property. Make it FixedSingle:

this.FormBorderStyle = FormBorderStyle.FixedSingle;

How do I list all tables in all databases in SQL Server in a single result set?

for a simple way to get all tables on the server, try this:

SET NOCOUNT ON
DECLARE @AllTables table (CompleteTableName nvarchar(4000))
INSERT INTO @AllTables (CompleteTableName)
    EXEC sp_msforeachdb 'select @@SERVERNAME+''.''+''?''+''.''+s.name+''.''+t.name from [?].sys.tables t inner join sys.schemas s on t.schema_id=s.schema_id'
SET NOCOUNT OFF
SELECT * FROM @AllTables ORDER BY 1

it will return a single column that contains the server+database+schema+table name: sample output:

CompleteTableName
--------------------------------------------
YourServer.YourDatabase1.YourSchema1.YourTable1
YourServer.YourDatabase1.YourSchema1.YourTable2
YourServer.YourDatabase1.YourSchema2.YourTable1
YourServer.YourDatabase1.YourSchema2.YourTable2
YourServer.YourDatabase2.YourSchema1.YourTable1

if you are not on SQL Server 2005 or up, replace the DECLARE @AllTables table with CREATE TABLE #AllTables and then every @AllTables with #AllTables and it will work.

EDIT
here is a version that will allow a search parameter to be used on any part or parts of the server+database+schema+table names:

SET NOCOUNT ON
DECLARE @AllTables table (CompleteTableName nvarchar(4000))
DECLARE @Search nvarchar(4000)
       ,@SQL   nvarchar(4000)
SET @Search=null --all rows
SET @SQL='select @@SERVERNAME+''.''+''?''+''.''+s.name+''.''+t.name from [?].sys.tables t inner join sys.schemas s on t.schema_id=s.schema_id WHERE @@SERVERNAME+''.''+''?''+''.''+s.name+''.''+t.name LIKE ''%'+ISNULL(@SEARCH,'')+'%'''

INSERT INTO @AllTables (CompleteTableName)
    EXEC sp_msforeachdb @SQL
SET NOCOUNT OFF
SELECT * FROM @AllTables ORDER BY 1

set @Search to NULL for all tables, set it to things like 'dbo.users' or 'users' or '.master.dbo' or even include wildcards like '.master.%.u', etc.

How to convert an array into an object using stdClass()

If you want to recursively convert the entire array into an Object type (stdClass) then , below is the best method and it's not time-consuming or memory deficient especially when you want to do a recursive (multi-level) conversion compared to writing your own function.

$array_object = json_decode(json_encode($array));

Java equivalent of unsigned long long?

The org.apache.axis.types package has a

UnsignedLong class.

for maven:

<dependency>
    <groupId>org.apache.axis</groupId>
    <artifactId>axis</artifactId>
    <version>1.4</version>
</dependency>

What is default session timeout in ASP.NET?

It is 20 Minutes according to MSDN

From MSDN:

Optional TimeSpan attribute.

Specifies the number of minutes a session can be idle before it is abandoned. The timeout attribute cannot be set to a value that is greater than 525,601 minutes (1 year) for the in-process and state-server modes. The session timeout configuration setting applies only to ASP.NET pages. Changing the session timeout value does not affect the session time-out for ASP pages. Similarly, changing the session time-out for ASP pages does not affect the session time-out for ASP.NET pages. The default is 20 minutes.

Can't ping a local VM from the host

On top of using a bridged connection, I had to turn on Find Devices and Content on the VM's Windows Server 2012 control panel network settings. Hope this helps somebody as none the other answers worked to ping the VM machine.

Could not find folder 'tools' inside SDK

I faced similar issue when the SDK tools installation was failed during the initial setup. To resolution is to download SDK tools from Android Developer Site

  • Expand "USE AN EXISTING IDE" section and download standalone SDK tools
  • Choose your destination as (%HOMEPATH%\android-sdks)
  • Now start Android-SDKs folder and run SDK manager

What's the difference between "git reset" and "git checkout"?

One simple use case when reverting change:
1. Use reset if you want to undo staging of a modified file.
2. Use checkout if you want to discard changes to unstaged file/s.

How to stop execution after a certain time in Java?

If you can't go over your time limit (it's a hard limit) then a thread is your best bet. You can use a loop to terminate the thread once you get to the time threshold. Whatever is going on in that thread at the time can be interrupted, allowing calculations to stop almost instantly. Here is an example:

Thread t = new Thread(myRunnable); // myRunnable does your calculations

long startTime = System.currentTimeMillis();
long endTime = startTime + 60000L;

t.start(); // Kick off calculations

while (System.currentTimeMillis() < endTime) {
    // Still within time theshold, wait a little longer
    try {
         Thread.sleep(500L);  // Sleep 1/2 second
    } catch (InterruptedException e) {
         // Someone woke us up during sleep, that's OK
    }
}

t.interrupt();  // Tell the thread to stop
t.join();       // Wait for the thread to cleanup and finish

That will give you resolution to about 1/2 second. By polling more often in the while loop, you can get that down.

Your runnable's run would look something like this:

public void run() {
    while (true) {
        try {
            // Long running work
            calculateMassOfUniverse();
        } catch (InterruptedException e) {
            // We were signaled, clean things up
            cleanupStuff();
            break;           // Leave the loop, thread will exit
    }
}

Update based on Dmitri's answer

Dmitri pointed out TimerTask, which would let you avoid the loop. You could just do the join call and the TimerTask you setup would take care of interrupting the thread. This would let you get more exact resolution without having to poll in a loop.

SQLSTATE[HY000] [1045] Access denied for user 'root'@'localhost' (using password: YES) symfony2

database_password: password would between quotes: " or '.

like so:

database_password: "password"

Reading file from Workspace in Jenkins with Groovy script

As mentioned in a different post Read .txt file from workspace groovy script in Jenkins I was struggling to make it work for the pom modules for a file in the workspace, in the Extended Choice Parameter. Here is my solution with the printlns:

import groovy.util.XmlSlurper
import java.util.Map
import jenkins.*
import jenkins.model.*
import hudson.*
import hudson.model.*    

try{
//get Jenkins instance
    def jenkins = Jenkins.instance
//get job Item
    def item = jenkins.getItemByFullName("The_JOB_NAME")
    println item
// get workspacePath for the job Item
    def workspacePath = jenkins.getWorkspaceFor (item)
    println workspacePath

    def file = new File(workspacePath.toString()+"\\pom.xml")
    def pomFile = new XmlSlurper().parse(file)
    def pomModules = pomFile.modules.children().join(",")
    return pomModules
} catch (Exception ex){
    println ex.message
}

WordPress - Check if user is logged in

Try following code that worked fine for me

global $current_user;
get_currentuserinfo();

Then, use following code to check whether user has logged in or not.

if ($current_user->ID == '') { 
    //show nothing to user
}
else { 
    //write code to show menu here
}

Extract every nth element of a vector

To select every nth element from any starting position in the vector

nth_element <- function(vector, starting_position, n) { 
  vector[seq(starting_position, length(vector), n)] 
  }

# E.g.
vec <- 1:12

nth_element(vec, 1, 3)
# [1]  1  4  7 10

nth_element(vec, 2, 3)
# [1]  2  5  8 11

Git Cherry-pick vs Merge Workflow

Rebase and Cherry-pick is the only way you can keep clean commit history. Avoid using merge and avoid creating merge conflict. If you are using gerrit set one project to Merge if necessary and one project to cherry-pick mode and try yourself.

How to reload a page using JavaScript

You can simply use

window.location=document.URL

where document.URL gets the current page URL and window.location reloads it.

How to have a a razor action link open in a new tab?

asp.net mvc ActionLink new tab with angular parameter

<a  target="_blank" class="btn" data-ng-href="@Url.Action("RunReport", "Performance")?hotelCode={{hotel.code}}">Select Room</a>

Creating a Jenkins environment variable using Groovy

You can also define a variable without the EnvInject Plugin within your Groovy System Script:

import hudson.model.*
def build = Thread.currentThread().executable
def pa = new ParametersAction([
  new StringParameterValue("FOO", "BAR")
])
build.addAction(pa)

Then you can access this variable in the next build step which (for example) is an windows batch command:

@echo off
Setlocal EnableDelayedExpansion
echo FOO=!FOO!

This echo will show you "FOO=BAR".

Regards

How to retrieve data from a SQL Server database in C#?

You can use this simple method after setting up your connection:

private void getAgentInfo(string key)//"key" is your search paramter inside database
    {
        con.Open();
        string sqlquery = "SELECT * FROM TableName WHERE firstname = @fName";

        SqlCommand command = new SqlCommand(sqlquery, con); 
        SqlDataReader sReader;

        command.Parameters.Clear();
        command.Parameters.AddWithValue("@fName", key);
        sReader = command.ExecuteReader();

        while (sReader.Read())
        {
            textBoxLastName.Text = sReader["Lastname"].ToString(); //SqlDataReader
            //["LastName"] the name of your column you want to retrieve from DB

            textBoxAge.Text = sReader["age"].ToString();
            //["age"] another column you want to retrieve
        }
        con.Close();
    }

Now you can pass the key to this method by your textBoxFirstName like:

getAgentInfo(textBoxFirstName.Text);

What is the purpose of "pip install --user ..."?

Best way to is install virtualenv and not require the --user confusion. You will get more flexibility and not worry about clobbering the different python versions and projects everytime you pip install a package.

https://virtualenv.pypa.io/en/stable/

linux script to kill java process

If you just want to kill any/all java processes, then all you need is;

killall java

If, however, you want to kill the wskInterface process in particular, then you're most of the way there, you just need to strip out the process id;

PID=`ps -ef | grep wskInterface | awk '{ print $2 }'`
kill -9 $PID

Should do it, there is probably an easier way though...

HEAD and ORIG_HEAD in Git

From man 7 gitrevisions:

HEAD names the commit on which you based the changes in the working tree. FETCH_HEAD records the branch which you fetched from a remote repository with your last git fetch invocation. ORIG_HEAD is created by commands that move your HEAD in a drastic way, to record the position of the HEAD before their operation, so that you can easily change the tip of the branch back to the state before you ran them. MERGE_HEAD records the commit(s) which you are merging into your branch when you run git merge. CHERRY_PICK_HEAD records the commit which you are cherry-picking when you run git cherry-pick.

Difference between <input type='submit' /> and <button type='submit'>text</button>

In summary :

<input type="submit">

<button type="submit"> Submit </button>

Both by default will visually draw a button that performs the same action (submit the form).

However, it is recommended to use <button type="submit"> because it has better semantics, better ARIA support and it is easier to style.

Find (and kill) process locking port 3000 on Mac

Find and kill:

This single command line is easy and works correctly.

kill -9 $(lsof -ti tcp:3000)

Setting background colour of Android layout element

The answers above all are static. I thought I would provide a dynamic answer. The two files that will need to be in sync are the relative foo.xml with the layout and activity_bar.java which corresponds to the Java class corresponding to this R.layout.foo.

In foo.xml set an id for the entire layout:

<?xml version="1.0" encoding="utf-8"?>
<RelativeLayout android:id="@+id/foo" .../>

And in activity_bar.java set the color in the onCreate():

public class activity_bar extends AppCompatActivty {

    @Override
    protected void onCreate(Bundle savedInstanceState) {
        super.onCreate(savedInstanceState);
        setContentView(R.layout.foo);

            //Set an id to the layout
        RelativeLayout currentLayout = 
                    (RelativeLayout) findViewById(R.id.foo);

        currentLayout.setBackgroundColor(Color.RED);
        ...
    }
    ...
}

I hope this helps.

Volatile Vs Atomic

So what will happen if two threads attack a volatile primitive variable at same time?

Usually each one can increment the value. However sometime, both will update the value at the same time and instead of incrementing by 2 total, both thread increment by 1 and only 1 is added.

Does this mean that whosoever takes lock on it, that will be setting its value first.

There is no lock. That is what synchronized is for.

And in if meantime, some other thread comes up and read old value while first thread was changing its value, then doesn't new thread will read its old value?

Yes,

What is the difference between Atomic and volatile keyword?

AtomicXxxx wraps a volatile so they are basically same, the difference is that it provides higher level operations such as CompareAndSwap which is used to implement increment.

AtomicXxxx also supports lazySet. This is like a volatile set, but doesn't stall the pipeline waiting for the write to complete. It can mean that if you read a value you just write you might see the old value, but you shouldn't be doing that anyway. The difference is that setting a volatile takes about 5 ns, bit lazySet takes about 0.5 ns.

what does it mean "(include_path='.:/usr/share/pear:/usr/share/php')"?

I had the same error while including file from root of my project in codeigniter.I was using this in common.php of my project.

<?php include_once base_url().'csrf-magic-master/csrf-magic.php';  ?>

i changed it to

<?php include_once ('csrf-magic-master/csrf-magic.php');  ?>

Working fine now.

Could not load file or assembly CrystalDecisions.ReportAppServer.ClientDoc

It turns out the answer was ridiculously simple, but mystifying as to why it was necessary.

In the IIS Manager on the server, I set the application pool for my web application to not allow 32-bit assemblies.

It seems it assumes, on a 64-bit system, that you must want the 32 bit assembly. Bizarre.

Find closest previous element jQuery

var link = $("#me").closest(":has(h3 span b)").find('h3 span b');

Example: http://jsfiddle.net/e27r8/

This uses the closest()[docs] method to get the first ancestor that has a nested h3 span b, then does a .find().

Of course you could have multiple matches.


Otherwise, you're looking at doing a more direct traversal.

var link = $("#me").closest("h3 + div").prev().find('span b');

edit: This one works with your updated HTML.

Example: http://jsfiddle.net/e27r8/2/


EDIT: Updated to deal with updated question.

var link = $("#me").closest("h3 + *").prev().find('span b');

This makes the targeted element for .closest() generic, so that even if there is no parent, it will still work.

Example: http://jsfiddle.net/e27r8/4/

T-SQL stored procedure that accepts multiple Id values

Try This One:

@list_of_params varchar(20) -- value 1, 2, 5, 7, 20 

SELECT d.[Name]
FROM Department d
where @list_of_params like ('%'+ CONVERT(VARCHAR(10),d.Id)  +'%')

very simple.

How to throw RuntimeException ("cannot find symbol")

Just for others: be sure it is new RuntimeException, not new RuntimeErrorException which needs error as an argument.

Empty or Null value display in SSRS text boxes

Either in SQL or in report code (as per adolf garlic's function suggestion)

At this moment in time, I'd do it in the report. I have very few reports against a busy OLTP server and an underwhelmed report server. If I had a different mix I'd do it in SQL.

Either way is acceptable...

Recreate the default website in IIS

You can try to restore your previous state by doing the following:

  1. Go to IIS Manager
  2. Right-click on your Local Computer.
  3. Point to All Tasks
  4. Point to Backup/Restore Configuration
  5. Select the configuration you want to restore
  6. Wait untill configuration applies

Pass Javascript Array -> PHP

You could use JSON.stringify(array) to encode your array in JavaScript, and then use $array=json_decode($_POST['jsondata']); in your PHP script to retrieve it.

How can I declare a global variable in Angular 2 / Typescript?

A shared service is the best approach

export class SharedService {
  globalVar:string;
}

But you need to be very careful when registering it to be able to share a single instance for whole your application. You need to define it when registering your application:

bootstrap(AppComponent, [SharedService]);

But not to define it again within the providers attributes of your components:

@Component({
  (...)
  providers: [ SharedService ], // No
  (...)
})

Otherwise a new instance of your service will be created for the component and its sub-components.

You can have a look at this question regarding how dependency injection and hierarchical injectors work in Angular 2:

You should notice that you can also define Observable properties in the service to notify parts of your application when your global properties change:

export class SharedService {
  globalVar:string;
  globalVarUpdate:Observable<string>;
  globalVarObserver:Observer;

  constructor() {
    this.globalVarUpdate = Observable.create((observer:Observer) => {
      this.globalVarObserver = observer;
    });
  }

  updateGlobalVar(newValue:string) {
    this.globalVar = newValue;
    this.globalVarObserver.next(this.globalVar);
  }
}

See this question for more details:

Oracle date "Between" Query

Judging from your output it looks like you have defined START_DATE as a timestamp. If it were a regular date Oracle would be able to handle the implicit conversion. But as it isn't you need to explicitly cast those strings to be dates.

SQL> alter session set nls_date_format = 'dd-mon-yyyy hh24:mi:ss'
  2  /

Session altered.

SQL>
SQL> select * from t23
  2  where start_date between '15-JAN-10' and '17-JAN-10'
  3  /

no rows selected

SQL> select * from t23
  2  where start_date between to_date('15-JAN-10') and to_date('17-JAN-10')
  3  /

WIDGET                          START_DATE
------------------------------  ----------------------
Small Widget                    15-JAN-10 04.25.32.000    

SQL> 

But we still only get one row. This is because START_DATE has a time element. If we don't specify the time component Oracle defaults it to midnight. That is fine for the from side of the BETWEEN but not for the until side:

SQL> select * from t23
  2  where start_date between to_date('15-JAN-10') 
  3                       and to_date('17-JAN-10 23:59:59')
  4  /

WIDGET                          START_DATE
------------------------------  ----------------------
Small Widget                    15-JAN-10 04.25.32.000
Product 1                       17-JAN-10 04.31.32.000

SQL>

edit

If you cannot pass in the time component there are a couple of choices. One is to change the WHERE clause to remove the time element from the criteria:

where trunc(start_date) between to_date('15-JAN-10') 
                            and to_date('17-JAN-10')

This might have an impact on performance, because it disqualifies any b-tree index on START_DATE. You would need to build a function-based index instead.

Alternatively you could add the time element to the date in your code:

where start_date between to_date('15-JAN-10') 
                     and to_date('17-JAN-10') + (86399/86400) 

Because of these problems many people prefer to avoid the use of between by checking for date boundaries like this:

where start_date >= to_date('15-JAN-10') 
and start_date < to_date('18-JAN-10')

Selenium WebDriver and DropDown Boxes

You can use this

(new SelectElement(driver.FindElement(By.Id(""))).SelectByText("");

How do you enable mod_rewrite on any OS?

Module rewrite_module is built-in in to the server most cases

Use .htaccess

Use the Mod Rewrite Generator at http://www.generateit.net/mod-rewrite/

Checking if a character is a special character in Java

You can use regular expressions.

String input = ...
if (input.matches("[^a-zA-Z0-9 ]"))

If your definition of a 'special character' is simply anything that doesn't apply to your other filters that you already have, then you can simply add an else. Also note that you have to use else if in this case:

if(c == ' ') {
    blankCount++;
} else if (Character.isDigit(c)) {
    digitCount++;
} else if (Character.isLetter(c)) {
    letterCount++;
} else { 
  specialcharCount++;
}

Showing empty view when ListView is empty

When you extend FragmentActivity or Activity and not ListActivity, you'll want to take a look at:

ListView.setEmptyView()

Environment variables in Jenkins

The environment variables displayed in Jenkins (Manage Jenkins -> System information) are inherited from the system (i.e. inherited environment variables)

If you run env command in a shell you should see the same environment variables as Jenkins shows.

These variables are either set by the shell/system or by you in ~/.bashrc, ~/.bash_profile.

There are also environment variables set by Jenkins when a job executes, but these are not displayed in the System Information.

Git: Find the most recent common ancestor of two branches

git diff master...feature

shows all the new commits of your current (possibly multi-commit) feature branch.

man git-diff documents that:

git diff A...B

is the same as:

git diff $(git merge-base A B) B

but the ... is easier to type and remember.

As mentioned by Dave, the special case of HEAD can be omitted. So:

git diff master...HEAD

is the same as:

git diff master...

which is enough if the current branch is feature.

Finally, remember that order matters! Doing git diff feature...master will show changes that are on master not on feature.

I wish more git commands would support that syntax, but I don't think they do. And some even have different semantics for ...: What are the differences between double-dot ".." and triple-dot "..." in Git commit ranges?

How can I get the full object in Node.js's console.log(), rather than '[Object]'?

You can also do

console.log(JSON.stringify(myObject, null, 3));

Select columns in PySpark dataframe

Use df.schema.names:

spark.version
# u'2.2.0'

df = spark.createDataFrame([("foo", 1), ("bar", 2)])
df.show()
# +---+---+ 
# | _1| _2|
# +---+---+
# |foo|  1| 
# |bar|  2|
# +---+---+

df.schema.names
# ['_1', '_2']

for i in df.schema.names:
  # df_new = df.withColumn(i, [do-something])
  print i
# _1
# _2

JNZ & CMP Assembly Instructions

JNZ     Jump if Not Zero    ZF=0

Indeed, this is confusing right.

To make it easier to understand, replace Not Zero with Not Set. (Please take note this is for your own understanding)

Hence,

JNZ     Jump if Not Set     ZF=0

Not Set means flag Z = 0. So Jump (Jump if Not Set)

Set means flag Z = 1. So, do NOT Jump

why does DateTime.ToString("dd/MM/yyyy") give me dd-MM-yyyy?

Dumb question/answer perhaps, but have you tried dd/MM/yyyy? Note the capitalization.

mm is for minutes with a leading zero. So I doubt that's what you want.

This may be helpful: http://www.geekzilla.co.uk/View00FF7904-B510-468C-A2C8-F859AA20581F.htm

accessing a docker container from another container

It's easy. If you have two or more running container, complete next steps:

docker network create myNetwork
docker network connect myNetwork web1
docker network connect myNetwork web2

Now you connect from web1 to web2 container or the other way round.

Use the internal network IP addresses which you can find by running:

docker network inspect myNetwork

Note that only internal IP addresses and ports are accessible to the containers connected by the network bridge.

So for example assuming that web1 container was started with: docker run -p 80:8888 web1 (meaning that its server is running on port 8888 internally), and inspecting myNetwork shows that web1's IP is 172.0.0.2, you can connect from web2 to web1 using curl 172.0.0.2:8888).

gitx How do I get my 'Detached HEAD' commits back into master

If checkout master was the last thing you did, then the reflog entry HEAD@{1} will contain your commits (otherwise use git reflog or git log -p to find them). Use git merge HEAD@{1} to fast forward them into master.

EDIT:

As noted in the comments, Git Ready has a great article on this.

git reflog and git reflog --all will give you the commit hashes of the mis-placed commits.

Git Ready: Reflog, Your Safety Net

Source: http://gitready.com/intermediate/2009/02/09/reflog-your-safety-net.html

How to delete an app from iTunesConnect / App Store Connect

Apps can’t be deleted if they are part of a Game Center group, in an app bundle, or currently displayed on a store. You’ll want to remove the app from sale or from the group if you want to delete it.

Source: iTunes Connect Developer Guide - Transferring and Deleting Apps

sklearn plot confusion matrix with labels

As hinted in this question, you have to "open" the lower-level artist API, by storing the figure and axis objects passed by the matplotlib functions you call (the fig, ax and cax variables below). You can then replace the default x- and y-axis ticks using set_xticklabels/set_yticklabels:

from sklearn.metrics import confusion_matrix

labels = ['business', 'health']
cm = confusion_matrix(y_test, pred, labels)
print(cm)
fig = plt.figure()
ax = fig.add_subplot(111)
cax = ax.matshow(cm)
plt.title('Confusion matrix of the classifier')
fig.colorbar(cax)
ax.set_xticklabels([''] + labels)
ax.set_yticklabels([''] + labels)
plt.xlabel('Predicted')
plt.ylabel('True')
plt.show()

Note that I passed the labels list to the confusion_matrix function to make sure it's properly sorted, matching the ticks.

This results in the following figure:

enter image description here

Pass variables between two PHP pages without using a form or the URL of page

Have you tried adding both to $_SESSION?

Then at the top of your page2.php just add:

<?php
session_start();

How to prevent scientific notation in R?

To set the use of scientific notation in your entire R session, you can use the scipen option. From the documentation (?options):

‘scipen’: integer.  A penalty to be applied when deciding to print
          numeric values in fixed or exponential notation.  Positive
          values bias towards fixed and negative towards scientific
          notation: fixed notation will be preferred unless it is more
          than ‘scipen’ digits wider.

So in essence this value determines how likely it is that scientific notation will be triggered. So to prevent scientific notation, simply use a large positive value like 999:

options(scipen=999)

Python NLTK: SyntaxError: Non-ASCII character '\xc3' in file (Sentiment Analysis -NLP)

Add the following to the top of your file # coding=utf-8

If you go to the link in the error you can seen the reason why:

Defining the Encoding

Python will default to ASCII as standard encoding if no other encoding hints are given. To define a source code encoding, a magic comment must be placed into the source files either as first or second line in the file, such as: # coding=

Provide schema while reading csv file as a dataframe

I'm using the solution provided by Arunakiran Nulu in my analysis (see the code). Despite it is able to assign the correct types to the columns, all the values returned are null. Previously, I've tried to the option .option("inferSchema", "true") and it returns the correct values in the dataframe (although different type).

val customSchema = StructType(Array(
    StructField("numicu", StringType, true),
    StructField("fecha_solicitud", TimestampType, true),
    StructField("codtecnica", StringType, true),
    StructField("tecnica", StringType, true),
    StructField("finexploracion", TimestampType, true),
    StructField("ultimavalidacioninforme", TimestampType, true),
    StructField("validador", StringType, true)))

val df_explo = spark.read
        .format("csv")
        .option("header", "true")
        .option("delimiter", "\t")
        .option("timestampFormat", "yyyy/MM/dd HH:mm:ss") 
        .schema(customSchema)
        .load(filename)

Result

root


|-- numicu: string (nullable = true)
 |-- fecha_solicitud: timestamp (nullable = true)
 |-- codtecnica: string (nullable = true)
 |-- tecnica: string (nullable = true)
 |-- finexploracion: timestamp (nullable = true)
 |-- ultimavalidacioninforme: timestamp (nullable = true)
 |-- validador: string (nullable = true)

and the table is:

|numicu|fecha_solicitud|codtecnica|tecnica|finexploracion|ultimavalidacioninforme|validador|
+------+---------------+----------+-------+--------------+-----------------------+---------+
|  null|           null|      null|   null|          null|                   null|     null|
|  null|           null|      null|   null|          null|                   null|     null|
|  null|           null|      null|   null|          null|                   null|     null|
|  null|           null|      null|   null|          null|                   null|     null|

How to pass parameter to click event in Jquery

As DOC says, you can pass data to the handler as next:

// say your selector and click handler looks something like this...
$("some selector").on('click',{param1: "Hello", param2: "World"}, cool_function);

// in your function, just grab the event object and go crazy...
function cool_function(event){
    alert(event.data.param1);
    alert(event.data.param2);

    // access element's id where click occur
    alert( event.target.id ); 
}

How to extract a value from a string using regex and a shell?

Yes regex can certainly be used to extract part of a string. Unfortunately different flavours of *nix and different tools use slightly different Regex variants.

This sed command should work on most flavours (Tested on OS/X and Redhat)

echo '12 BBQ ,45 rofl, 89 lol' | sed  's/^.*,\([0-9][0-9]*\).*$/\1/g'

How to use android emulator for testing bluetooth application?

You can't. The emulator does not support Bluetooth, as mentioned in the SDK's docs and several other places. Android emulator does not have bluetooth capabilities".

You can only use real devices.

Emulator Limitations

The functional limitations of the emulator include:

  • No support for placing or receiving actual phone calls. However, You can simulate phone calls (placed and received) through the emulator console
  • No support for USB
  • No support for device-attached headphones
  • No support for determining SD card insert/eject
  • No support for WiFi, Bluetooth, NFC

Refer to the documentation

Insert line after first match using sed

Try doing this using GNU sed:

sed '/CLIENTSCRIPT="foo"/a CLIENTSCRIPT2="hello"' file

if you want to substitute in-place, use

sed -i '/CLIENTSCRIPT="foo"/a CLIENTSCRIPT2="hello"' file

Output

CLIENTSCRIPT="foo"
CLIENTSCRIPT2="hello"
CLIENTFILE="bar"

Doc

  • see sed doc and search \a (append)

How to have a drop down <select> field in a rails form?

<%= f.select :email_provider, ["gmail","yahoo","msn"]%>

How to list the properties of a JavaScript object?

Building on the accepted answer.

If the Object has properties you want to call say .properties() try!

var keys = Object.keys(myJSONObject);

for (var j=0; j < keys.length; j++) {
  Object[keys[j]].properties();
}

Regular expression to allow spaces between words

tl;dr

Just add a space in your character class.

^[a-zA-Z0-9_ ]*$

 


Now, if you want to be strict...

The above isn't exactly correct. Due to the fact that * means zero or more, it would match all of the following cases that one would not usually mean to match:

  • An empty string, "".
  • A string comprised entirely of spaces, "      ".
  • A string that leads and / or trails with spaces, "   Hello World  ".
  • A string that contains multiple spaces in between words, "Hello   World".

Originally I didn't think such details were worth going into, as OP was asking such a basic question that it seemed strictness wasn't a concern. Now that the question's gained some popularity however, I want to say...

...use @stema's answer.

Which, in my flavor (without using \w) translates to:

^[a-zA-Z0-9_]+( [a-zA-Z0-9_]+)*$

(Please upvote @stema regardless.)

Some things to note about this (and @stema's) answer:

  • If you want to allow multiple spaces between words (say, if you'd like to allow accidental double-spaces, or if you're working with copy-pasted text from a PDF), then add a + after the space:

    ^\w+( +\w+)*$
    
  • If you want to allow tabs and newlines (whitespace characters), then replace the space with a \s+:

    ^\w+(\s+\w+)*$
    

    Here I suggest the + by default because, for example, Windows linebreaks consist of two whitespace characters in sequence, \r\n, so you'll need the + to catch both.

Still not working?

Check what dialect of regular expressions you're using.* In languages like Java you'll have to escape your backslashes, i.e. \\w and \\s. In older or more basic languages and utilities, like sed, \w and \s aren't defined, so write them out with character classes, e.g. [a-zA-Z0-9_] and [\f\n\p\r\t], respectively.

 


* I know this question is tagged , but based on 25,000+ views, I'm guessing it's not only those folks who are coming across this question. Currently it's the first hit on google for the search phrase, regular expression space word.

Summarizing count and conditional aggregate functions on the same factor

Assuming that your original dataset is similar to the one you created (i.e. with NA as character. You could specify na.strings while reading the data using read.table. But, I guess NAs would be detected automatically.

The price column is factor which needs to be converted to numeric class. When you use as.numeric, all the non-numeric elements (i.e. "NA", FALSE) gets coerced to NA) with a warning.

library(dplyr)
df %>%
     mutate(price=as.numeric(as.character(price))) %>%  
     group_by(company, year, product) %>%
     summarise(total.count=n(), 
               count=sum(is.na(price)), 
               avg.price=mean(price,na.rm=TRUE),
               max.price=max(price, na.rm=TRUE))

data

I am using the same dataset (except the ... row) that was showed.

df = tbl_df(data.frame(company=c("Acme", "Meca", "Emca", "Acme", "Meca","Emca"),
 year=c("2011", "2010", "2009", "2011", "2010", "2013"), product=c("Wrench", "Hammer",
 "Sonic Screwdriver", "Fairy Dust", "Kindness", "Helping Hand"), price=c("5.67",
 "7.12", "12.99", "10.99", "NA",FALSE)))

Best way to check function arguments?

One way is to use assert:

def myFunction(a,b,c):
    "This is an example function I'd like to check arguments of"
    assert isinstance(a, int), 'a should be an int'
    # or if you want to allow whole number floats: assert int(a) == a
    assert b > 0 and b < 10, 'b should be betwen 0 and 10'
    assert isinstance(c, str) and c, 'c should be a non-empty string'

How to get Wikipedia content using Wikipedia's API?

You can download the Wikipedia database directly and parse all pages to XML with Wiki Parser, which is a standalone application. The first paragraph is a separate node in the resulting XML.

Alternatively, you can extract the first paragraph from its plain-text output.

Oracle Partition - Error ORA14400 - inserted partition key does not map to any partition

For this issue need to add the partition for date column values, If last partition 20201231245959, then inserting the 20210110245959 values, this issue will occurs.

For that need to add the 2021 partition into that table

ALTER TABLE TABLE_NAME ADD PARTITION PARTITION_NAME VALUES LESS THAN (TO_DATE('2021-12-31 24:59:59', 'SYYYY-MM-DD HH24:MI:SS', 'NLS_CALENDAR=GREGORIAN')) NOCOMPRESS

How to display an unordered list in two columns?

You can do this really easily with the jQuery-Columns Plugin for example to split a ul with a class of .mylist you would do

$('.mylist').cols(2);

Here's a live example on jsfiddle

I like this better than with CSS because with the CSS solution not everything aligns vertically to the top.

nuget 'packages' element is not declared warning

This happens because VS doesn't know the schema of this file. Note that this file is more of an implementation detail, and not something you normally need to open directly. Instead, you can use the NuGet dialog to manage the packages installed in a project.

How to find count of Null and Nan values for each column in a PySpark dataframe efficiently?

An alternative to the already provided ways is to simply filter on the column like so

df = df.where(F.col('columnNameHere').isNull())

This has the added benefit that you don't have to add another column to do the filtering and it's quick on larger data sets.

Importing modules from parent folder

You can use OS depending path in "module search path" which is listed in sys.path . So you can easily add parent directory like following

import sys
sys.path.insert(0,'..')

If you want to add parent-parent directory,

sys.path.insert(0,'../..')

This works both in python 2 and 3.

Where is JAVA_HOME on macOS Mojave (10.14) to Lion (10.7)?

Got the same issue after I upgrade my Mac OS and following worked for me:

  1. cmd>vi ~/.bash_profile

  2. Add/update the line for JAVA_HOME: export JAVA_HOME="/Library/Java/JavaVirtualMachines/jdk1.7.0_15.jdk/Contents/Home"

  3. cmd>source ~/.bash_profile or open a new terminal

I think the jdk version might differ, so just use the version which you have under /Library/Java/JavaVirtualMachines/

Difference between request.getSession() and request.getSession(true)

A major practical difference is its use:

in security scenario where we always needed a new session, we should use request.getSession(true).

request.getSession(false): will return null if no session found.

How do I set a conditional breakpoint in gdb, when char* x points to a string whose value equals "hello"?

You can use strcmp:

break x:20 if strcmp(y, "hello") == 0

20 is line number, x can be any filename and y can be any variable.

Do we need type="text/css" for <link> in HTML5

The HTML5 spec says that the type attribute is purely advisory and explains in detail how browsers should act if it's omitted (too much to quote here). It doesn't explicitly say that an omitted type attribute is either valid or invalid, but you can safely omit it knowing that browsers will still react as you expect.

How to check version of a CocoaPods framework

You can figure out version of Cocoapods by using below command :

pod —-version

o/p : 1.2.1

Now if you want detailed version of Gems and Cocoapods then use below command :

gem which cocoapods (without sudo)

o/p : /Library/Ruby/Gems/2.0.0/gems/cocoapods-1.2.1/lib/cocoapods.rb

sudo gem which cocoapods (with sudo)

o/p : /Library/Ruby/Gems/2.0.0/gems/cocoapods-1.2.1/lib/cocoapods.rb

Screenshot 1

Now if you want to get specific version of Pod present in Podfile then simply use command pod install in terminal. This will show list of pod being used in project along with version.

Screenshot 2

What is the difference between char s[] and char *s?

In the light of comments here it should be obvious that : char * s = "hello" ; Is a bad idea, and should be used in very narrow scope.

This might be a good opportunity to point out that "const correctness" is a "good thing". Whenever and wherever You can, use the "const" keyword to protect your code, from "relaxed" callers or programmers, which are usually most "relaxed" when pointers come into play.

Enough melodrama, here is what one can achieve when adorning pointers with "const". (Note: One has to read pointer declarations right-to-left.) Here are the 3 different ways to protect yourself when playing with pointers :

const DBJ* p means "p points to a DBJ that is const" 

— that is, the DBJ object can't be changed via p.

DBJ* const p means "p is a const pointer to a DBJ" 

— that is, you can change the DBJ object via p, but you can't change the pointer p itself.

const DBJ* const p means "p is a const pointer to a const DBJ" 

— that is, you can't change the pointer p itself, nor can you change the DBJ object via p.

The errors related to attempted const-ant mutations are caught at compile time. There is no runtime space or speed penalty for const.

(Assumption is you are using C++ compiler, of course ?)

--DBJ

How to use GNU Make on Windows?

Here's how I got it to work:

  copy c:\MinGW\bin\mingw32-make.exe c:\MinGW\bin\make.exe

Then I am able to open a command prompt and type make:

  C:\Users\Dell>make
  make: *** No targets specified and no makefile found.  Stop.

Which means it's working now!

find filenames NOT ending in specific extensions on Unix?

$ find . -name \*.exe -o -name \*.dll -o -print

The first two -name options have no -print option, so they skipped. Everything else is printed.

Add to Array jQuery

push is a native javascript method. You could use it like this:

var array = [1, 2, 3];
array.push(4); // array now is [1, 2, 3, 4]
array.push(5, 6, 7); // array now is [1, 2, 3, 4, 5, 6, 7]

getting the index of a row in a pandas apply function

Either:

1. with row.name inside the apply(..., axis=1) call:

df = pandas.DataFrame([[1,2,3],[4,5,6]], columns=['a','b','c'], index=['x','y'])

   a  b  c
x  1  2  3
y  4  5  6

df.apply(lambda row: row.name, axis=1)

x    x
y    y

2. with iterrows() (slower)

DataFrame.iterrows() allows you to iterate over rows, and access their index:

for idx, row in df.iterrows():
    ...

How do you delete an ActiveRecord object?

There is delete, delete_all, destroy, and destroy_all.

The docs are: older docs and Rails 3.0.0 docs

delete doesn't instantiate the objects, while destroy does. In general, delete is faster than destroy.

MySQL pivot table query with dynamic columns

I have a slightly different way of doing this than the accepted answer. This way you can avoid using GROUP_CONCAT which has a limit of 1024 characters and will not work if you have a lot of fields.

SET @sql = '';
SELECT
    @sql := CONCAT(@sql,if(@sql='','',', '),temp.output)
FROM
(
    SELECT
      DISTINCT
        CONCAT(
         'MAX(IF(pa.fieldname = ''',
          fieldname,
          ''', pa.fieldvalue, NULL)) AS ',
          fieldname
        ) as output
    FROM
        product_additional
) as temp;

SET @sql = CONCAT('SELECT p.id
                    , p.name
                    , p.description, ', @sql, ' 
                   FROM product p
                   LEFT JOIN product_additional AS pa 
                    ON p.id = pa.id
                   GROUP BY p.id');

PREPARE stmt FROM @sql;
EXECUTE stmt;
DEALLOCATE PREPARE stmt;

How to properly ignore exceptions

try:
      doSomething()
except Exception: 
    pass
else:
      stuffDoneIf()
      TryClauseSucceeds()

FYI the else clause can go after all exceptions and will only be run if the code in the try doesn't cause an exception.

What is the iPhone 4 user-agent?

You can use:

http://whatsmyuseragent.com/

To find your user agent (Google: "What is my user agent" gives this answer)

Format date with Moment.js

Include moment.js and using the below code you can format your date

var formatDate= 1399919400000;

var responseDate = moment(formatDate).format('DD/MM/YYYY');

My output is "13/05/2014"

How to limit google autocomplete results to City and Country only

<html>
  <head>
    <title>Example Using Google Complete API</title>   
  </head>
  <body>

<form>
   <input id="geocomplete" type="text" placeholder="Type an address/location"/>
    </form>
   <script src="http://maps.googleapis.com/maps/api/js?sensor=false&amp;libraries=places"></script>
   <script src="http://ajax.googleapis.com/ajax/libs/jquery/1.7.1/jquery.min.js"></script>

  <script src="http://ubilabs.github.io/geocomplete/jquery.geocomplete.js"></script>
  <script>
   $(function(){        
    $("#geocomplete").geocomplete();                           
   });
  </script>
 </body>
</html>

For more information visit this link

How to insert text in a td with id, using JavaScript

If your <td> is not empty, one popular trick is to insert a non breaking space &nbsp; in it, such that:

 <td id="td1">&nbsp;</td>

Then you will be able to use:

 document.getElementById('td1').firstChild.data = 'New Value';

Otherwise, if you do not fancy adding the meaningless &nbsp you can use the solution that Jonathan Fingland described in the other answer.

In JavaScript, why is "0" equal to false, but when tested by 'if' it is not false by itself?

Your quotes around the 0 make it a string, which is evaluated as true.

Remove the quotes and it should work.

if (0) console.log("ha") 

In CSS what is the difference between "." and "#" when declaring a set of styles?

.class targets the following element:

<div class="class"></div>

#class targets the following element:

<div id="class"></div>

Note that the id MUST be unique throughout the document, whilst any number of elements may share a class.

How to get current instance name from T-SQL

I found this:

EXECUTE xp_regread
        @rootkey = 'HKEY_LOCAL_MACHINE',
        @key = 'SOFTWARE\Microsoft\Microsoft SQL Server',
        @value_name = 'InstalledInstances'

That will give you list of all instances installed in your server.


The ServerName property of the SERVERPROPERTY function and @@SERVERNAME return similar information. The ServerName property provides the Windows server and instance name that together make up the unique server instance. @@SERVERNAME provides the currently configured local server name.

And Microsoft example for current server is:

SELECT CONVERT(sysname, SERVERPROPERTY('servername'));

This scenario is useful when there are multiple instances of SQL Server installed on a Windows server, and the client must open another connection to the same instance used by the current connection.

How to modify PATH for Homebrew?

open bash profile in textEdit

open -e .bash_profile

Edit file or paste in front of PATH export PATH=/usr/bin:/usr/sbin:/bin:/sbin:/usr/local/bin:/usr/local/sbin:~/bin

save & close the file

*To open .bash_profile directly open textEdit > file > recent

How should I tackle --secure-file-priv in MySQL?

If the file is local to your machine use the LOCAL in your command

LOAD DATA LOCAL INFILE "text.txt" INTO table mytable;

What does "res.render" do, and what does the html file look like?

Renders a view and sends the rendered HTML string to the client.

res.render('index');

Or

res.render('index', function(err, html) {
  if(err) {...}
  res.send(html);
});

DOCS HERE: https://expressjs.com/en/api.html#res.render

What is the difference between __dirname and ./ in node.js?

./ refers to the current working directory, except in the require() function. When using require(), it translates ./ to the directory of the current file called. __dirname is always the directory of the current file.

For example, with the following file structure

/home/user/dir/files/config.json

{
  "hello": "world"
}

/home/user/dir/files/somefile.txt

text file

/home/user/dir/dir.js

var fs = require('fs');

console.log(require('./files/config.json'));
console.log(fs.readFileSync('./files/somefile.txt', 'utf8'));

If I cd into /home/user/dir and run node dir.js I will get

{ hello: 'world' }
text file

But when I run the same script from /home/user/ I get

{ hello: 'world' }

Error: ENOENT, no such file or directory './files/somefile.txt'
    at Object.openSync (fs.js:228:18)
    at Object.readFileSync (fs.js:119:15)
    at Object.<anonymous> (/home/user/dir/dir.js:4:16)
    at Module._compile (module.js:432:26)
    at Object..js (module.js:450:10)
    at Module.load (module.js:351:31)
    at Function._load (module.js:310:12)
    at Array.0 (module.js:470:10)
    at EventEmitter._tickCallback (node.js:192:40)

Using ./ worked with require but not for fs.readFileSync. That's because for fs.readFileSync, ./ translates into the cwd (in this case /home/user/). And /home/user/files/somefile.txt does not exist.

How do I check if file exists in Makefile so I can delete it?

Or just put it on one line, as make likes it:

if [ -a myApp ]; then rm myApp; fi;

Insert using LEFT JOIN and INNER JOIN

INSERT INTO Test([col1],[col2]) (
    SELECT 
        a.Name AS [col1],
        b.sub AS [col2] 
    FROM IdTable b 
    INNER JOIN Nametable a ON b.no = a.no
)

Color different parts of a RichTextBox string

Selecting text as said from somebody, may the selection appear momentarily. In Windows Forms applications there is no other solutions for the problem, but today I found a bad, working, way to solve: you can put a PictureBox in overlapping to the RichtextBox with the screenshot of if, during the selection and the changing color or font, making it after reappear all, when the operation is complete.

Code is here...

//The PictureBox has to be invisible before this, at creation
//tb variable is your RichTextBox
//inputPreview variable is your PictureBox
using (Graphics g = inputPreview.CreateGraphics())
{
    Point loc = tb.PointToScreen(new Point(0, 0));
    g.CopyFromScreen(loc, loc, tb.Size);
    Point pt = tb.GetPositionFromCharIndex(tb.TextLength);
    g.FillRectangle(new SolidBrush(Color.Red), new Rectangle(pt.X, 0, 100, tb.Height));
}
inputPreview.Invalidate();
inputPreview.Show();
//Your code here (example: tb.Select(...); tb.SelectionColor = ...;)
inputPreview.Hide();

Better is to use WPF; this solution isn't perfect, but for Winform it works.

How to determine CPU and memory consumption from inside a process?

Windows

Some of the above values are easily available from the appropriate WIN32 API, I just list them here for completeness. Others, however, need to be obtained from the Performance Data Helper library (PDH), which is a bit "unintuitive" and takes a lot of painful trial and error to get to work. (At least it took me quite a while, perhaps I've been only a bit stupid...)

Note: for clarity all error checking has been omitted from the following code. Do check the return codes...!


  • Total Virtual Memory:

    #include "windows.h"
    
    MEMORYSTATUSEX memInfo;
    memInfo.dwLength = sizeof(MEMORYSTATUSEX);
    GlobalMemoryStatusEx(&memInfo);
    DWORDLONG totalVirtualMem = memInfo.ullTotalPageFile;
    

    Note: The name "TotalPageFile" is a bit misleading here. In reality this parameter gives the "Virtual Memory Size", which is size of swap file plus installed RAM.

  • Virtual Memory currently used:

    Same code as in "Total Virtual Memory" and then

     DWORDLONG virtualMemUsed = memInfo.ullTotalPageFile - memInfo.ullAvailPageFile;
    
  • Virtual Memory currently used by current process:

    #include "windows.h"
    #include "psapi.h"
    
    PROCESS_MEMORY_COUNTERS_EX pmc;
    GetProcessMemoryInfo(GetCurrentProcess(), (PROCESS_MEMORY_COUNTERS*)&pmc, sizeof(pmc));
    SIZE_T virtualMemUsedByMe = pmc.PrivateUsage;
    



  • Total Physical Memory (RAM):

    Same code as in "Total Virtual Memory" and then

    DWORDLONG totalPhysMem = memInfo.ullTotalPhys;
    
  • Physical Memory currently used:

    Same code as in "Total Virtual Memory" and then

    DWORDLONG physMemUsed = memInfo.ullTotalPhys - memInfo.ullAvailPhys;
    
  • Physical Memory currently used by current process:

    Same code as in "Virtual Memory currently used by current process" and then

    SIZE_T physMemUsedByMe = pmc.WorkingSetSize;
    



  • CPU currently used:

    #include "TCHAR.h"
    #include "pdh.h"
    
    static PDH_HQUERY cpuQuery;
    static PDH_HCOUNTER cpuTotal;
    
    void init(){
        PdhOpenQuery(NULL, NULL, &cpuQuery);
        // You can also use L"\\Processor(*)\\% Processor Time" and get individual CPU values with PdhGetFormattedCounterArray()
        PdhAddEnglishCounter(cpuQuery, L"\\Processor(_Total)\\% Processor Time", NULL, &cpuTotal);
        PdhCollectQueryData(cpuQuery);
    }
    
    double getCurrentValue(){
        PDH_FMT_COUNTERVALUE counterVal;
    
        PdhCollectQueryData(cpuQuery);
        PdhGetFormattedCounterValue(cpuTotal, PDH_FMT_DOUBLE, NULL, &counterVal);
        return counterVal.doubleValue;
    }
    
  • CPU currently used by current process:

    #include "windows.h"
    
    static ULARGE_INTEGER lastCPU, lastSysCPU, lastUserCPU;
    static int numProcessors;
    static HANDLE self;
    
    void init(){
        SYSTEM_INFO sysInfo;
        FILETIME ftime, fsys, fuser;
    
        GetSystemInfo(&sysInfo);
        numProcessors = sysInfo.dwNumberOfProcessors;
    
        GetSystemTimeAsFileTime(&ftime);
        memcpy(&lastCPU, &ftime, sizeof(FILETIME));
    
        self = GetCurrentProcess();
        GetProcessTimes(self, &ftime, &ftime, &fsys, &fuser);
        memcpy(&lastSysCPU, &fsys, sizeof(FILETIME));
        memcpy(&lastUserCPU, &fuser, sizeof(FILETIME));
    }
    
    double getCurrentValue(){
        FILETIME ftime, fsys, fuser;
        ULARGE_INTEGER now, sys, user;
        double percent;
    
        GetSystemTimeAsFileTime(&ftime);
        memcpy(&now, &ftime, sizeof(FILETIME));
    
        GetProcessTimes(self, &ftime, &ftime, &fsys, &fuser);
        memcpy(&sys, &fsys, sizeof(FILETIME));
        memcpy(&user, &fuser, sizeof(FILETIME));
        percent = (sys.QuadPart - lastSysCPU.QuadPart) +
            (user.QuadPart - lastUserCPU.QuadPart);
        percent /= (now.QuadPart - lastCPU.QuadPart);
        percent /= numProcessors;
        lastCPU = now;
        lastUserCPU = user;
        lastSysCPU = sys;
    
        return percent * 100;
    }
    

Linux

On Linux the choice that seemed obvious at first was to use the POSIX APIs like getrusage() etc. I spent some time trying to get this to work, but never got meaningful values. When I finally checked the kernel sources themselves, I found out that apparently these APIs are not yet completely implemented as of Linux kernel 2.6!?

In the end I got all values via a combination of reading the pseudo-filesystem /proc and kernel calls.

  • Total Virtual Memory:

    #include "sys/types.h"
    #include "sys/sysinfo.h"
    
    struct sysinfo memInfo;
    
    sysinfo (&memInfo);
    long long totalVirtualMem = memInfo.totalram;
    //Add other values in next statement to avoid int overflow on right hand side...
    totalVirtualMem += memInfo.totalswap;
    totalVirtualMem *= memInfo.mem_unit;
    
  • Virtual Memory currently used:

    Same code as in "Total Virtual Memory" and then

    long long virtualMemUsed = memInfo.totalram - memInfo.freeram;
    //Add other values in next statement to avoid int overflow on right hand side...
    virtualMemUsed += memInfo.totalswap - memInfo.freeswap;
    virtualMemUsed *= memInfo.mem_unit;
    
  • Virtual Memory currently used by current process:

    #include "stdlib.h"
    #include "stdio.h"
    #include "string.h"
    
    int parseLine(char* line){
        // This assumes that a digit will be found and the line ends in " Kb".
        int i = strlen(line);
        const char* p = line;
        while (*p <'0' || *p > '9') p++;
        line[i-3] = '\0';
        i = atoi(p);
        return i;
    }
    
    int getValue(){ //Note: this value is in KB!
        FILE* file = fopen("/proc/self/status", "r");
        int result = -1;
        char line[128];
    
        while (fgets(line, 128, file) != NULL){
            if (strncmp(line, "VmSize:", 7) == 0){
                result = parseLine(line);
                break;
            }
        }
        fclose(file);
        return result;
    }
    



  • Total Physical Memory (RAM):

    Same code as in "Total Virtual Memory" and then

    long long totalPhysMem = memInfo.totalram;
    //Multiply in next statement to avoid int overflow on right hand side...
    totalPhysMem *= memInfo.mem_unit;
    
  • Physical Memory currently used:

    Same code as in "Total Virtual Memory" and then

    long long physMemUsed = memInfo.totalram - memInfo.freeram;
    //Multiply in next statement to avoid int overflow on right hand side...
    physMemUsed *= memInfo.mem_unit;
    
  • Physical Memory currently used by current process:

    Change getValue() in "Virtual Memory currently used by current process" as follows:

    int getValue(){ //Note: this value is in KB!
        FILE* file = fopen("/proc/self/status", "r");
        int result = -1;
        char line[128];
    
        while (fgets(line, 128, file) != NULL){
            if (strncmp(line, "VmRSS:", 6) == 0){
                result = parseLine(line);
                break;
            }
        }
        fclose(file);
        return result;
    }
    



  • CPU currently used:

    #include "stdlib.h"
    #include "stdio.h"
    #include "string.h"
    
    static unsigned long long lastTotalUser, lastTotalUserLow, lastTotalSys, lastTotalIdle;
    
    void init(){
        FILE* file = fopen("/proc/stat", "r");
        fscanf(file, "cpu %llu %llu %llu %llu", &lastTotalUser, &lastTotalUserLow,
            &lastTotalSys, &lastTotalIdle);
        fclose(file);
    }
    
    double getCurrentValue(){
        double percent;
        FILE* file;
        unsigned long long totalUser, totalUserLow, totalSys, totalIdle, total;
    
        file = fopen("/proc/stat", "r");
        fscanf(file, "cpu %llu %llu %llu %llu", &totalUser, &totalUserLow,
            &totalSys, &totalIdle);
        fclose(file);
    
        if (totalUser < lastTotalUser || totalUserLow < lastTotalUserLow ||
            totalSys < lastTotalSys || totalIdle < lastTotalIdle){
            //Overflow detection. Just skip this value.
            percent = -1.0;
        }
        else{
            total = (totalUser - lastTotalUser) + (totalUserLow - lastTotalUserLow) +
                (totalSys - lastTotalSys);
            percent = total;
            total += (totalIdle - lastTotalIdle);
            percent /= total;
            percent *= 100;
        }
    
        lastTotalUser = totalUser;
        lastTotalUserLow = totalUserLow;
        lastTotalSys = totalSys;
        lastTotalIdle = totalIdle;
    
        return percent;
    }
    
  • CPU currently used by current process:

    #include "stdlib.h"
    #include "stdio.h"
    #include "string.h"
    #include "sys/times.h"
    #include "sys/vtimes.h"
    
    static clock_t lastCPU, lastSysCPU, lastUserCPU;
    static int numProcessors;
    
    void init(){
        FILE* file;
        struct tms timeSample;
        char line[128];
    
        lastCPU = times(&timeSample);
        lastSysCPU = timeSample.tms_stime;
        lastUserCPU = timeSample.tms_utime;
    
        file = fopen("/proc/cpuinfo", "r");
        numProcessors = 0;
        while(fgets(line, 128, file) != NULL){
            if (strncmp(line, "processor", 9) == 0) numProcessors++;
        }
        fclose(file);
    }
    
    double getCurrentValue(){
        struct tms timeSample;
        clock_t now;
        double percent;
    
        now = times(&timeSample);
        if (now <= lastCPU || timeSample.tms_stime < lastSysCPU ||
            timeSample.tms_utime < lastUserCPU){
            //Overflow detection. Just skip this value.
            percent = -1.0;
        }
        else{
            percent = (timeSample.tms_stime - lastSysCPU) +
                (timeSample.tms_utime - lastUserCPU);
            percent /= (now - lastCPU);
            percent /= numProcessors;
            percent *= 100;
        }
        lastCPU = now;
        lastSysCPU = timeSample.tms_stime;
        lastUserCPU = timeSample.tms_utime;
    
        return percent;
    }
    

TODO: Other Platforms

I would assume, that some of the Linux code also works for the Unixes, except for the parts that read the /proc pseudo-filesystem. Perhaps on Unix these parts can be replaced by getrusage() and similar functions? If someone with Unix know-how could edit this answer and fill in the details?!

How does @synchronized lock/unlock in Objective-C?

In Objective-C, a @synchronized block handles locking and unlocking (as well as possible exceptions) automatically for you. The runtime dynamically essentially generates an NSRecursiveLock that is associated with the object you're synchronizing on. This Apple documentation explains it in more detail. This is why you're not seeing the log messages from your NSLock subclass — the object you synchronize on can be anything, not just an NSLock.

Basically, @synchronized (...) is a convenience construct that streamlines your code. Like most simplifying abstractions, it has associated overhead (think of it as a hidden cost), and it's good to be aware of that, but raw performance is probably not the supreme goal when using such constructs anyway.

Declaring variable workbook / Worksheet vba

Third solution: I would set ws to a sheet of workbook wb as the use of Sheet("name") always refers to the active workbook, which might change as your code develops.

sub kl() 

    Dim wb As Workbook
    Dim ws As Worksheet

    Set wb = ActiveWorkbook
    'be aware as this might produce an error, if Shet "name" does not exist
    Set ws = wb.Sheets("name")
    ' if wb is other than the active workbook
    wb.activate
    ws.Select

End Sub

How to Add Date Picker To VBA UserForm

Just throw some light in to some issues related to this control.

Date picker is not a standard control that comes with office package. So developers encountered issues like missing date picker controls when application deployed in some other machiens/versions of office. In order to use it you have to activate the reference to the .dll, .ocx file that contains it.

In the event of a missing date picker, you have to replace MSCOMCT2.OCX file in System or System32 directory and register it properly. Try this link to do the proper replacement of the file.

In the VBA editor menu bar-> select tools-> references and then find the date picker reference and check it.

If you need the file, download MSCOMCT2.OCX from here.

Lookup City and State by Zip Google Geocode Api

function getCityState($zip, $blnUSA = true) {
    $url = "http://maps.googleapis.com/maps/api/geocode/json?address=" . $zip . "&sensor=true";

    $address_info = file_get_contents($url);
    $json = json_decode($address_info);
    $city = "";
    $state = "";
    $country = "";
    if (count($json->results) > 0) {
        //break up the components
        $arrComponents = $json->results[0]->address_components;

        foreach($arrComponents as $index=>$component) {
            $type = $component->types[0];

            if ($city == "" && ($type == "sublocality_level_1" || $type == "locality") ) {
                $city = trim($component->short_name);
            }
            if ($state == "" && $type=="administrative_area_level_1") {
                $state = trim($component->short_name);
            }
            if ($country == "" && $type=="country") {
                $country = trim($component->short_name);

                if ($blnUSA && $country!="US") {
                    $city = "";
                    $state = "";
                    break;
                }
            }
            if ($city != "" && $state != "" && $country != "") {
                //we're done
                break;
            }
        }
    }
    $arrReturn = array("city"=>$city, "state"=>$state, "country"=>$country);

    die(json_encode($arrReturn));
}

php exec() is not executing the command

I already said that I was new to exec() function. After doing some more digging, I came upon 2>&1 which needs to be added at the end of command in exec().

Thanks @mattosmat for pointing it out in the comments too. I did not try this at once because you said it is a Linux command, I am on Windows.

So, what I have discovered, the command is actually executing in the back-end. That is why I could not see it actually running, which I was expecting to happen.

For all of you, who had similar problem, my advise is to use that command. It will point out all the errors and also tell you info/details about execution.

exec('some_command 2>&1', $output);
print_r($output);  // to see the response to your command

Thanks for all the help guys, I appreciate it ;)

Installing Numpy on 64bit Windows 7 with Python 2.7.3

Try the (unofficial) binaries in this site:

http://www.lfd.uci.edu/~gohlke/pythonlibs/#numpy

You can get the newest numpy x64 with or without Intel MKL libs for Python 2.7 or Python 3.

How to set order of repositories in Maven settings.xml

As far as I know, the order of the repositories in your pom.xml will also decide the order of the repository access.

As for configuring repositories in settings.xml, I've read that the order of repositories is interestingly enough the inverse order of how the repositories will be accessed.

Here a post where someone explains this curiosity:
http://community.jboss.org/message/576851

How to capitalize the first letter of text in a TextView in an Android Application

The accepted answer is good, but if you are using it to get values from a textView in android, it would be good to check if the string is empty. If the string is empty it would throw an exception.

private String capitizeString(String name){
    String captilizedString="";
    if(!name.trim().equals("")){
       captilizedString = name.substring(0,1).toUpperCase() + name.substring(1);
    }
    return captilizedString;
}

How to use DISTINCT and ORDER BY in same SELECT statement?

Try next, but it's not useful for huge data...

SELECT DISTINCT Cat FROM (
  SELECT Category as Cat FROM MonitoringJob ORDER BY CreationDate DESC
);

SQL Server: Filter output of sp_who2

Slight improvement to Astander's answer. I like to put my criteria at top, and make it easier to reuse day to day:

DECLARE @Spid INT, @Status VARCHAR(MAX), @Login VARCHAR(MAX), @HostName VARCHAR(MAX), @BlkBy VARCHAR(MAX), @DBName VARCHAR(MAX), @Command VARCHAR(MAX), @CPUTime INT, @DiskIO INT, @LastBatch VARCHAR(MAX), @ProgramName VARCHAR(MAX), @SPID_1 INT, @REQUESTID INT

    --SET @SPID = 10
    --SET @Status = 'BACKGROUND'
    --SET @LOGIN = 'sa'
    --SET @HostName = 'MSSQL-1'
    --SET @BlkBy = 0
    --SET @DBName = 'master'
    --SET @Command = 'SELECT INTO'
    --SET @CPUTime = 1000
    --SET @DiskIO = 1000
    --SET @LastBatch = '10/24 10:00:00'
    --SET @ProgramName = 'Microsoft SQL Server Management Studio - Query'
    --SET @SPID_1 = 10
    --SET @REQUESTID = 0

    SET NOCOUNT ON 
    DECLARE @Table TABLE(
            SPID INT,
            Status VARCHAR(MAX),
            LOGIN VARCHAR(MAX),
            HostName VARCHAR(MAX),
            BlkBy VARCHAR(MAX),
            DBName VARCHAR(MAX),
            Command VARCHAR(MAX),
            CPUTime INT,
            DiskIO INT,
            LastBatch VARCHAR(MAX),
            ProgramName VARCHAR(MAX),
            SPID_1 INT,
            REQUESTID INT
    )
    INSERT INTO @Table EXEC sp_who2
    SET NOCOUNT OFF
    SELECT  *
    FROM    @Table
    WHERE
    (@Spid IS NULL OR SPID = @Spid)
    AND (@Status IS NULL OR Status = @Status)
    AND (@Login IS NULL OR Login = @Login)
    AND (@HostName IS NULL OR HostName = @HostName)
    AND (@BlkBy IS NULL OR BlkBy = @BlkBy)
    AND (@DBName IS NULL OR DBName = @DBName)
    AND (@Command IS NULL OR Command = @Command)
    AND (@CPUTime IS NULL OR CPUTime >= @CPUTime)
    AND (@DiskIO IS NULL OR DiskIO >= @DiskIO)
    AND (@LastBatch IS NULL OR LastBatch >= @LastBatch)
    AND (@ProgramName IS NULL OR ProgramName = @ProgramName)
    AND (@SPID_1 IS NULL OR SPID_1 = @SPID_1)
    AND (@REQUESTID IS NULL OR REQUESTID = @REQUESTID)

Python regex to match dates

Well, from my understanding, simply for matching this format in a given string, I prefer this regular expression:

pattern='[0-9|/]+'

to match the format in a more strict way, the following works:

pattern='(?:[0-9]{2}/){2}[0-9]{2}'

Personally, I cannot agree with unutbu's answer since sometimes we use regular expression for "finding" and "extract", not only "validating".

What is a web service endpoint?

This is a shorter and hopefully clearer answer... Yes, the endpoint is the URL where your service can be accessed by a client application. The same web service can have multiple endpoints, for example in order to make it available using different protocols.

Setting up MySQL and importing dump within Dockerfile

What I did was download my sql dump in a "db-dump" folder, and mounted it:

mysql:
 image: mysql:5.6
 environment:
   MYSQL_ROOT_PASSWORD: pass
 ports:
   - 3306:3306
 volumes:
   - ./db-dump:/docker-entrypoint-initdb.d

When I run docker-compose up for the first time, the dump is restored in the db.

How to call a method daily, at specific time, in C#?

I created a simple scheduler that is easy to use and you do not need to use external library. TaskScheduler is a singleton that keeps references on the timers so timers will not be garbage collected, it can schedule multiple tasks. You can set the first run (hour and minute), if at the time of scheduling this time is over scheduling start on the next day this at that time. But it is easy to customize the code.

Scheduling a new task is so simple. Example: At 11:52 the first task is for every 15 secunds, the second example is for every 5 secunds. For daily execution set 24 to the 3 parameter.

TaskScheduler.Instance.ScheduleTask(11, 52, 0.00417, 
    () => 
    {
        Debug.WriteLine("task1: " + DateTime.Now);
        //here write the code that you want to schedule
    });

TaskScheduler.Instance.ScheduleTask(11, 52, 0.00139,
    () =>
    {
        Debug.WriteLine("task2: " + DateTime.Now);
        //here write the code that you want to schedule
    });

My debug window:

task2: 07.06.2017 11:52:00
task1: 07.06.2017 11:52:00
task2: 07.06.2017 11:52:05
task2: 07.06.2017 11:52:10
task1: 07.06.2017 11:52:15
task2: 07.06.2017 11:52:15
task2: 07.06.2017 11:52:20
task2: 07.06.2017 11:52:25
...

Just add this class to your project:

public class TaskScheduler
{
    private static TaskScheduler _instance;
    private List<Timer> timers = new List<Timer>();

    private TaskScheduler() { }

    public static TaskScheduler Instance => _instance ?? (_instance = new TaskScheduler());

    public void ScheduleTask(int hour, int min, double intervalInHour, Action task)
    {
        DateTime now = DateTime.Now;
        DateTime firstRun = new DateTime(now.Year, now.Month, now.Day, hour, min, 0, 0);
        if (now > firstRun)
        {
            firstRun = firstRun.AddDays(1);
        }

        TimeSpan timeToGo = firstRun - now;
        if (timeToGo <= TimeSpan.Zero)
        {
            timeToGo = TimeSpan.Zero;
        }

        var timer = new Timer(x =>
        {
            task.Invoke();
        }, null, timeToGo, TimeSpan.FromHours(intervalInHour));

        timers.Add(timer);
    }
}

INNER JOIN vs LEFT JOIN performance in SQL Server

There is one important scenario that can lead to an outer join being faster than an inner join that has not been discussed yet.

When using an outer join, the optimizer is always free to drop the outer joined table from the execution plan if the join columns are the PK of the outer table, and none of the outer table columns are referenced outside of the outer join itself. For example SELECT A.* FROM A LEFT OUTER JOIN B ON A.KEY=B.KEY and B.KEY is the PK for B. Both Oracle (I believe I was using release 10) and Sql Server (I used 2008 R2) prune table B from the execution plan.

The same is not necessarily true for an inner join: SELECT A.* FROM A INNER JOIN B ON A.KEY=B.KEY may or may not require B in the execution plan depending on what constraints exist.

If A.KEY is a nullable foreign key referencing B.KEY, then the optimizer cannot drop B from the plan because it must confirm that a B row exists for every A row.

If A.KEY is a mandatory foreign key referencing B.KEY, then the optimizer is free to drop B from the plan because the constraints guarantee the existence of the row. But just because the optimizer can drop the table from the plan, doesn't mean it will. SQL Server 2008 R2 does NOT drop B from the plan. Oracle 10 DOES drop B from the plan. It is easy to see how the outer join will out-perform the inner join on SQL Server in this case.

This is a trivial example, and not practical for a stand-alone query. Why join to a table if you don't need to?

But this could be a very important design consideration when designing views. Frequently a "do-everything" view is built that joins everything a user might need related to a central table. (Especially if there are naive users doing ad-hoc queries that do not understand the relational model) The view may include all the relevent columns from many tables. But the end users might only access columns from a subset of the tables within the view. If the tables are joined with outer joins, then the optimizer can (and does) drop the un-needed tables from the plan.

It is critical to make sure that the view using outer joins gives the correct results. As Aaronaught has said - you cannot blindly substitute OUTER JOIN for INNER JOIN and expect the same results. But there are times when it can be useful for performance reasons when using views.

One last note - I haven't tested the impact on performance in light of the above, but in theory it seems you should be able to safely replace an INNER JOIN with an OUTER JOIN if you also add the condition <FOREIGN_KEY> IS NOT NULL to the where clause.

Generate JSON string from NSDictionary in iOS

public func jsonPrint(_ o: NSObject, spacing: String = "", after: String = "", before: String = "") {
    let newSpacing = spacing + "    "
    if o.isArray() {
        print(before + "[")
        if let a = o as? Array<NSObject> {
            for object in a {
                jsonPrint(object, spacing: newSpacing, after: object == a.last! ? "" : ",", before: newSpacing)
            }
        }
        print(spacing + "]" + after)
    } else {
        if o.isDictionary() {
            print(before + "{")
            if let a = o as? Dictionary<NSObject, NSObject> {
                for (key, val) in a {
                    jsonPrint(val, spacing: newSpacing, after: ",", before: newSpacing + key.description + " = ")
                }
            }
            print(spacing + "}" + after)
        } else {
            print(before + o.description + after)
        }
    }
}

This one is pretty close to original Objective-C print style

Proper MIME type for OTF fonts

One way to silence this warning from Chrome would be to update Chrome and then make sure your mime type is one of these:

 "font/ttf"
 "font/opentype"
 "application/font-woff"
 "application/x-font-type1"
 "application/x-font-ttf"
 "application/x-truetype-font"

This list is per the patch found at Bug 111418 at webkit.org.

The same patch demotes the message from a "Warning" to a "Log", so just upgrading Chrome to any post March-2013 version would get rid of the yellow triangle.

Since the question is about silencing a Chrome warning, and folks might be holding on to old Chrome versions for whatever reasons, I figured this was worth adding.

How do I get the value of a registry key and ONLY the value using powershell

If you create an object, you get a more readable output and also gain an object with properties you can access:

$path = 'HKLM:\SOFTWARE\Wow6432Node\Microsoft\.NETFramework'
$obj  = New-Object -TypeName psobject

Get-Item -Path $path | Select-Object -ExpandProperty Property | Sort | % {
$command = [String]::Format('(Get-ItemProperty -Path "{0}" -Name "{1}")."{1}"', $path, $_)
$value = Invoke-Expression -Command $command
$obj | Add-Member -MemberType NoteProperty -Name $_ -Value $value}

Write-Output $obj | fl

Sample output: InstallRoot : C:\Windows\Microsoft.NET\Framework\

And the object: $obj.InstallRoot = C:\Windows\Microsoft.NET\Framework\

The truth of the matter is this is way more complicated than it needs to be. Here is a much better example, and much simpler:

$path = 'HKLM:\SOFTWARE\Wow6432Node\Microsoft\.NETFramework'
$objReg = Get-ItemProperty -Path $path | Select -Property *

$objReg is now a custom object where each registry entry is a property name. You can view the formatted list via:

write-output $objReg

InstallRoot        : C:\Windows\Microsoft.NET\Framework\
DbgManagedDebugger : "C:\windows\system32\vsjitdebugger.exe"

And you have access to the object itself:

$objReg.InstallRoot
C:\Windows\Microsoft.NET\Framework\

Rails Root directory path?

In Rails 3 and newer:

Rails.root

which returns a Pathname object. If you want a string you have to add .to_s. If you want another path in your Rails app, you can use join like this:

Rails.root.join('app', 'assets', 'images', 'logo.png')

In Rails 2 you can use the RAILS_ROOT constant, which is a string.

How do I change the UUID of a virtual disk?

Though you have solved the problem, I just post the reason here for some others with the similar problem.

The reason is there's an space in your path(directory name VirtualBox VMs) which will separate the command. So the error appears.

Binding a list in @RequestParam

Just complementing what Donal Fellows said, you can use List with @RequestParam

public String controllerMethod(@RequestParam(value="myParam") List<ObjectToParse> myParam){
....
}

Hope it helps!

Difference between jQuery’s .hide() and setting CSS to display: none

.hide() stores the previous display property just before setting it to none, so if it wasn't the standard display property for the element you're a bit safer, .show() will use that stored property as what to go back to. So...it does some extra work, but unless you're doing tons of elements, the speed difference should be negligible.

Classes cannot be accessed from outside package

Do you by any chance have two PUBLICclass classes in your project, where one is public (the one of which you posted the signature here), and another one which is package visible, and you import the wrong one in your code ?

Disabling SSL Certificate Validation in Spring RestTemplate

@Bean
public RestTemplate restTemplate() 
                throws KeyStoreException, NoSuchAlgorithmException, KeyManagementException {
    TrustStrategy acceptingTrustStrategy = (X509Certificate[] chain, String authType) -> true;

    SSLContext sslContext = org.apache.http.ssl.SSLContexts.custom()
                    .loadTrustMaterial(null, acceptingTrustStrategy)
                    .build();

    SSLConnectionSocketFactory csf = new SSLConnectionSocketFactory(sslContext);

    CloseableHttpClient httpClient = HttpClients.custom()
                    .setSSLSocketFactory(csf)
                    .build();

    HttpComponentsClientHttpRequestFactory requestFactory =
                    new HttpComponentsClientHttpRequestFactory();

    requestFactory.setHttpClient(httpClient);
    RestTemplate restTemplate = new RestTemplate(requestFactory);
    return restTemplate;
 }

How to disable clicking inside div

If you want it in pure CSS:

pointer-events:none;

C#: easiest way to populate a ListBox from a List

You can also use the AddRange method

listBox1.Items.AddRange(myList.ToArray());

Link vs compile vs controller

this is a good sample for understand directive phases http://codepen.io/anon/pen/oXMdBQ?editors=101

var app = angular.module('myapp', [])

app.directive('slngStylePrelink', function() {
    return {
        scope: {
            drctvName: '@'
        },
        controller: function($scope) {
            console.log('controller for ', $scope.drctvName);
        },
        compile: function(element, attr) {
            console.log("compile for ", attr.name)
            return {
                post: function($scope, element, attr) {
                    console.log('post link for ', attr.name)
                },
                pre: function($scope, element, attr) {
                    $scope.element = element;
                    console.log('pre link for ', attr.name)
                        // from angular.js 1.4.1
                    function ngStyleWatchAction(newStyles, oldStyles) {
                        if (oldStyles && (newStyles !== oldStyles)) {
                            forEach(oldStyles, function(val, style) {
                                element.css(style, '');
                            });
                        }
                        if (newStyles) element.css(newStyles);
                    }

                    $scope.$watch(attr.slngStylePrelink, ngStyleWatchAction, true);

                    // Run immediately, because the watcher's first run is async
                    ngStyleWatchAction($scope.$eval(attr.slngStylePrelink));
                }
            };
        }
    };
});

html

<body ng-app="myapp">
    <div slng-style-prelink="{height:'500px'}" drctv-name='parent' style="border:1px solid" name="parent">
        <div slng-style-prelink="{height:'50%'}" drctv-name='child' style="border:1px solid red" name='child'>
        </div>
    </div>
</body>

What is the best way to get the first letter from a string in Java, returned as a string of length 1?

import org.openjdk.jmh.annotations.Benchmark;
import org.openjdk.jmh.annotations.BenchmarkMode;
import org.openjdk.jmh.annotations.Fork;
import org.openjdk.jmh.annotations.Measurement;
import org.openjdk.jmh.annotations.Mode;
import org.openjdk.jmh.annotations.OutputTimeUnit;
import org.openjdk.jmh.annotations.Scope;
import org.openjdk.jmh.annotations.Setup;
import org.openjdk.jmh.annotations.State;
import org.openjdk.jmh.annotations.Warmup;

import java.util.concurrent.TimeUnit;

@State(Scope.Thread)
@BenchmarkMode(Mode.AverageTime)
@OutputTimeUnit(TimeUnit.NANOSECONDS)
@Warmup(iterations = 5, time = 1)
@Fork(value = 1)
@Measurement(iterations = 5, time = 1)
public class StringFirstCharBenchmark {

    private String source;

    @Setup
    public void init() {
        source = "MALE";
    }

    @Benchmark
    public String substring() {
        return source.substring(0, 1);
    }

    @Benchmark
    public String indexOf() {
        return String.valueOf(source.indexOf(0));
    }
}

Results:

+----------------------------------------------------------------------+
| Benchmark                           Mode  Cnt   Score   Error  Units |
+----------------------------------------------------------------------+
| StringFirstCharBenchmark.indexOf    avgt    5  23.777 ? 5.788  ns/op |
| StringFirstCharBenchmark.substring  avgt    5  11.305 ? 1.411  ns/op |
+----------------------------------------------------------------------+

What is the difference between find(), findOrFail(), first(), firstOrFail(), get(), list(), toArray()

  1. find($id) takes an id and returns a single model. If no matching model exist, it returns null.

  2. findOrFail($id) takes an id and returns a single model. If no matching model exist, it throws an error1.

  3. first() returns the first record found in the database. If no matching model exist, it returns null.

  4. firstOrFail() returns the first record found in the database. If no matching model exist, it throws an error1.

  5. get() returns a collection of models matching the query.

  6. pluck($column) returns a collection of just the values in the given column. In previous versions of Laravel this method was called lists.

  7. toArray() converts the model/collection into a simple PHP array.


Note: a collection is a beefed up array. It functions similarly to an array, but has a lot of added functionality, as you can see in the docs.

Unfortunately, PHP doesn't let you use a collection object everywhere you can use an array. For example, using a collection in a foreach loop is ok, put passing it to array_map is not. Similarly, if you type-hint an argument as array, PHP won't let you pass it a collection. Starting in PHP 7.1, there is the iterable typehint, which can be used to accept both arrays and collections.

If you ever want to get a plain array from a collection, call its all() method.


1 The error thrown by the findOrFail and firstOrFail methods is a ModelNotFoundException. If you don't catch this exception yourself, Laravel will respond with a 404, which is what you want most of the time.

How do I suspend painting for a control and its children?

The following is the same solution of ng5000 but doesn't use P/Invoke.

public static class SuspendUpdate
{
    private const int WM_SETREDRAW = 0x000B;

    public static void Suspend(Control control)
    {
        Message msgSuspendUpdate = Message.Create(control.Handle, WM_SETREDRAW, IntPtr.Zero,
            IntPtr.Zero);

        NativeWindow window = NativeWindow.FromHandle(control.Handle);
        window.DefWndProc(ref msgSuspendUpdate);
    }

    public static void Resume(Control control)
    {
        // Create a C "true" boolean as an IntPtr
        IntPtr wparam = new IntPtr(1);
        Message msgResumeUpdate = Message.Create(control.Handle, WM_SETREDRAW, wparam,
            IntPtr.Zero);

        NativeWindow window = NativeWindow.FromHandle(control.Handle);
        window.DefWndProc(ref msgResumeUpdate);

        control.Invalidate();
    }
}

How to test an Internet connection with bash?

This bash script continuously check for Internet and make a beep sound when the Internet is available.

#!/bin/bash
play -n synth 0.3 sine 800 vol 0.75
while :
do
pingtime=$(ping -w 1 8.8.8.8 | grep ttl)
if [ "$pingtime" = "" ] 
then 
   pingtimetwo=$(ping -w 1 www.google.com | grep ttl) 
   if [ "$pingtimetwo" = "" ] 
   then 
       clear ; echo 'Offline'
   else
       clear ; echo 'Online' ; play -n synth 0.3 sine 800 vol 0.75
   fi 
else
    clear ; echo 'Online' ; play -n synth 0.3 sine 800 vol 0.75
fi
sleep 1
done

Formatting Decimal places in R

You can try my package formattable.

> # devtools::install_github("renkun-ken/formattable")
> library(formattable)
> x <- formattable(1.128347132904321674821, digits = 2, format = "f")
> x
[1] 1.13

The good thing is, x is still a numeric vector and you can do more calculations with the same formatting.

> x + 1
[1] 2.13

Even better, the digits are not lost, you can reformat with more digits any time :)

> formattable(x, digits = 6, format = "f")
[1] 1.128347

Why does CreateProcess give error 193 (%1 is not a valid Win32 app)

Let me add an example here:

I'm trying to build Alluxio on windows platform and got the same issue, it's because the pom.xml contains below step:

      <plugin>
        <artifactId>exec-maven-plugin</artifactId>
        <groupId>org.codehaus.mojo</groupId>
        <inherited>false</inherited>
        <executions>
          <execution>
            <id>Check that there are no Windows line endings</id>
            <phase>compile</phase>
            <goals>
              <goal>exec</goal>
            </goals>
            <configuration>
              <executable>${build.path}/style/check_no_windows_line_endings.sh</executable>
            </configuration>
          </execution>
        </executions>
      </plugin>

The .sh file is not executable on windows so the error throws.

Comment it out if you do want build Alluxio on windows.

Console app arguments, how arguments are passed to Main method

in visual studio you can also do like that to pass simply or avoiding from comandline argument

 static void Main(string[] args)
    {
        if (args == null)
        {
            Console.WriteLine("args is null"); // Check for null array
        }
        else
        {
            args=new string[2];
            args[0] = "welcome in";
            args[1] = "www.overflow.com";
            Console.Write("args length is ");
            Console.WriteLine(args.Length); // Write array length
            for (int i = 0; i < args.Length; i++) // Loop through array
            {
                string argument = args[i];
                Console.Write("args index ");
                Console.Write(i); // Write index
                Console.Write(" is [");
                Console.Write(argument); // Write string
                Console.WriteLine("]");
            }
        }

How to draw a checkmark / tick using CSS?

After some changing to above Henry's answer, I got a tick with in a circle, I came here looking for that, so adding my code here.

_x000D_
_x000D_
.snackbar_circle {
  background-color: #f0f0f0;
  border-radius: 13px;
  padding: 0 5px;
}

.checkmark {
  font-family: arial;
  font-weight: bold;
  -ms-transform: scaleX(-1) rotate(-35deg);
  -webkit-transform: scaleX(-1) rotate(-35deg);
  transform: scaleX(-1) rotate(-35deg);
  color: #63BA3D;
  display: inline-block;
}
_x000D_
<span class="snackbar_circle">
   <span class="checkmark">L</span>
</span>
_x000D_
_x000D_
_x000D_

How do I revert a Git repository to a previous commit?

For rollback (or to revert):

1. git revert --no-commit "commit-code-to-remove" HEAD
(e.g. git revert --no-commit d57a39d HEAD)
2. git commit
3. git push

Try the above two steps, and if you find this is what you want then git push.

If you find something wrong, do:

git revert --abort

Generate a random number in a certain range in MATLAB

ocw.mit.edu is a great resource that has helped me a bunch. randi is the best option, but if your into number fun try using the floor function with rand to get what you want.

I drew a number line and came up with

floor(rand*8) + 13

"register" keyword in C?

Storytime!

C, as a language, is an abstraction of a computer. It allows you to do things, in terms of what a computer does, that is manipulate memory, do math, print things, etc.

But C is only an abstraction. And ultimately, what it's extracting from you is Assembly language. Assembly is the language that a CPU reads, and if you use it, you do things in terms of the CPU. What does a CPU do? Basically, it reads from memory, does math, and writes to memory. The CPU doesn't just do math on numbers in memory. First, you have to move a number from memory to memory inside the CPU called a register. Once you're done doing whatever you need to do to this number, you can move it back to normal system memory. Why use system memory at all? Registers are limited in number. You only get about a hundred bytes in modern processors, and older popular processors were even more fantastically limited (The 6502 had 3 8-bit registers for your free use). So, your average math operation looks like:

load first number from memory
load second number from memory
add the two
store answer into memory

A lot of that is... not math. Those load and store operations can take up to half your processing time. C, being an abstraction of computers, freed the programmer the worry of using and juggling registers, and since the number and type vary between computers, C places the responsibility of register allocation solely on the compiler. With one exception.

When you declare a variable register, you are telling the compiler "Yo, I intend for this variable to be used a lot and/or be short lived. If I were you, I'd try to keep it in a register." When the C standard says compilers don't have to actually do anything, that's because the C standard doesn't know what computer you're compiling for, and it might be like the 6502 above, where all 3 registers are needed just to operate, and there's no spare register to keep your number. However, when it says you can't take the address, that's because registers don't have addresses. They're the processor's hands. Since the compiler doesn't have to give you an address, and since it can't have an address at all ever, several optimizations are now open to the compiler. It could, say, keep the number in a register always. It doesn't have to worry about where it's stored in computer memory (beyond needing to get it back again). It could even pun it into another variable, give it to another processor, give it a changing location, etc.

tl;dr: Short-lived variables that do lots of math. Don't declare too many at once.

What is the best practice for creating a favicon on a web site?

I used https://iconifier.net I uploaded my image, downloaded images zip file, added images to my server, followed the directions on the site including adding the links to my index.html and it worked. My favicon now shows on my iPhone in Safari when 'Add to home screen'

How to create/read/write JSON files in Qt5

An example on how to use that would be great. There is a couple of examples at the Qt forum, but you're right that the official documentation should be expanded.

QJsonDocument on its own indeed doesn't produce anything, you will have to add the data to it. That's done through the QJsonObject, QJsonArray and QJsonValue classes. The top-level item needs to be either an array or an object (because 1 is not a valid json document, while {foo: 1} is.)

Can't get Gulp to run: cannot find module 'gulp-util'

I had the same issue, although the module that it was downloading was different. The only resolution to the problem is run the below command again:

npm install

What is the difference between \r and \n?

In C and C++, \n is a concept, \r is a character, and \r\n is (almost always) a portability bug.

Think of an old teletype. The print head is positioned on some line and in some column. When you send a printable character to the teletype, it prints the character at the current position and moves the head to the next column. (This is conceptually the same as a typewriter, except that typewriters typically moved the paper with respect to the print head.)

When you wanted to finish the current line and start on the next line, you had to do two separate steps:

  1. move the print head back to the beginning of the line, then
  2. move it down to the next line.

ASCII encodes these actions as two distinct control characters:

  • \x0D (CR) moves the print head back to the beginning of the line. (Unicode encodes this as U+000D CARRIAGE RETURN.)
  • \x0A (LF) moves the print head down to the next line. (Unicode encodes this as U+000A LINE FEED.)

In the days of teletypes and early technology printers, people actually took advantage of the fact that these were two separate operations. By sending a CR without following it by a LF, you could print over the line you already printed. This allowed effects like accents, bold type, and underlining. Some systems overprinted several times to prevent passwords from being visible in hardcopy. On early serial CRT terminals, CR was one of the ways to control the cursor position in order to update text already on the screen.

But most of the time, you actually just wanted to go to the next line. Rather than requiring the pair of control characters, some systems allowed just one or the other. For example:

  • Unix variants (including modern versions of Mac) use just a LF character to indicate a newline.
  • Old (pre-OSX) Macintosh files used just a CR character to indicate a newline.
  • VMS, CP/M, DOS, Windows, and many network protocols still expect both: CR LF.
  • Old IBM systems that used EBCDIC standardized on NL--a character that doesn't even exist in the ASCII character set. In Unicode, NL is U+0085 NEXT LINE, but the actual EBCDIC value is 0x15.

Why did different systems choose different methods? Simply because there was no universal standard. Where your keyboard probably says "Enter", older keyboards used to say "Return", which was short for Carriage Return. In fact, on a serial terminal, pressing Return actually sends the CR character. If you were writing a text editor, it would be tempting to just use that character as it came in from the terminal. Perhaps that's why the older Macs used just CR.

Now that we have standards, there are more ways to represent line breaks. Although extremely rare in the wild, Unicode has new characters like:

  • U+2028 LINE SEPARATOR
  • U+2029 PARAGRAPH SEPARATOR

Even before Unicode came along, programmers wanted simple ways to represent some of the most useful control codes without worrying about the underlying character set. C has several escape sequences for representing control codes:

  • \a (for alert) which rings the teletype bell or makes the terminal beep
  • \f (for form feed) which moves to the beginning of the next page
  • \t (for tab) which moves the print head to the next horizontal tab position

(This list is intentionally incomplete.)

This mapping happens at compile-time--the compiler sees \a and puts whatever magic value is used to ring the bell.

Notice that most of these mnemonics have direct correlations to ASCII control codes. For example, \a would map to 0x07 BEL. A compiler could be written for a system that used something other than ASCII for the host character set (e.g., EBCDIC). Most of the control codes that had specific mnemonics could be mapped to control codes in other character sets.

Huzzah! Portability!

Well, almost. In C, I could write printf("\aHello, World!"); which rings the bell (or beeps) and outputs a message. But if I wanted to then print something on the next line, I'd still need to know what the host platform requires to move to the next line of output. CR LF? CR? LF? NL? Something else? So much for portability.

C has two modes for I/O: binary and text. In binary mode, whatever data is sent gets transmitted as-is. But in text mode, there's a run-time translation that converts a special character to whatever the host platform needs for a new line (and vice versa).

Great, so what's the special character?

Well, that's implementation dependent, too, but there's an implementation-independent way to specify it: \n. It's typically called the "newline character".

This is a subtle but important point: \n is mapped at compile time to an implementation-defined character value which (in text mode) is then mapped again at run time to the actual character (or sequence of characters) required by the underlying platform to move to the next line.

\n is different than all the other backslash literals because there are two mappings involved. This two-step mapping makes \n significantly different than even \r, which is simply a compile-time mapping to CR (or the most similar control code in whatever the underlying character set is).

This trips up many C and C++ programmers. If you were to poll 100 of them, at least 99 will tell you that \n means line feed. This is not entirely true. Most (perhaps all) C and C++ implementations use LF as the magic intermediate value for \n, but that's an implementation detail. It's feasible for a compiler to use a different value. In fact, if the host character set is not a superset of ASCII (e.g., if it's EBCDIC), then \n will almost certainly not be LF.

So, in C and C++:

  • \r is literally a carriage return.
  • \n is a magic value that gets translated (in text mode) at run-time to/from the host platform's newline semantics.
  • \r\n is almost always a portability bug. In text mode, this gets translated to CR followed by the platform's newline sequence--probably not what's intended. In binary mode, this gets translated to CR followed by some magic value that might not be LF--possibly not what's intended.
  • \x0A is the most portable way to indicate an ASCII LF, but you only want to do that in binary mode. Most text-mode implementations will treat that like \n.

Can enums be subclassed to add new elements?

I suggest you take the other way around approach.

Instead of extending the existing enumeration, create a larger one and create a subset of it. For exemple if you had an enumeration called PET and you wanted to extend it to ANIMAL you should do this instead:

public enum ANIMAL {
    WOLF,CAT, DOG
} 
EnumSet<ANIMAL> pets = EnumSet.of(ANIMAL.CAT, ANIMAL.DOG);

Be careful, pets is not an immutable collections, you might want to use Guava or Java9 for more safety.

How to fix height of TR?

Putting div inside a td made it work for me.

<table width="100%">
    <tr><td><div style="font-size:2px; height:2px; vertical-align:middle;">&nbsp;</div></td></tr>

Changing one character in a string

Strings are immutable in Python, which means you cannot change the existing string. But if you want to change any character in it, you could create a new string out it as follows,

def replace(s, position, character):
    return s[:position] + character + s[position+1:]

replace('King', 1, 'o')
// result: Kong

Note: If you give the position value greater than the length of the string, it will append the character at the end.

replace('Dog', 10, 's')
// result: Dogs

How to make a variadic macro (variable number of arguments)

#define DEBUG

#ifdef DEBUG
  #define PRINT print
#else
  #define PRINT(...) ((void)0) //strip out PRINT instructions from code
#endif 

void print(const char *fmt, ...) {

    va_list args;
    va_start(args, fmt);
    vsprintf(str, fmt, args);
        va_end(args);

        printf("%s\n", str);

}

int main() {
   PRINT("[%s %d, %d] Hello World", "March", 26, 2009);
   return 0;
}

If the compiler does not understand variadic macros, you can also strip out PRINT with either of the following:

#define PRINT //

or

#define PRINT if(0)print

The first comments out the PRINT instructions, the second prevents PRINT instruction because of a NULL if condition. If optimization is set, the compiler should strip out never executed instructions like: if(0) print("hello world"); or ((void)0);

How do I measure a time interval in C?

If your Linux system supports it, clock_gettime(CLOCK_MONOTONIC) should be a high resolution timer that is unaffected by system date changes (e.g. NTP daemons).

How to get a reference to an iframe's window object inside iframe's onload handler created from parent window

You're declaring everything in the parent page. So the references to window and document are to the parent page's. If you want to do stuff to the iframe's, use iframe || iframe.contentWindow to access its window, and iframe.contentDocument || iframe.contentWindow.document to access its document.

There's a word for what's happening, possibly "lexical scope": What is lexical scope?

The only context of a scope is this. And in your example, the owner of the method is doc, which is the iframe's document. Other than that, anything that's accessed in this function that uses known objects are the parent's (if not declared in the function). It would be a different story if the function were declared in a different place, but it's declared in the parent page.

This is how I would write it:

(function () {
  var dom, win, doc, where, iframe;

  iframe = document.createElement('iframe');
  iframe.src = "javascript:false";

  where = document.getElementsByTagName('script')[0];
  where.parentNode.insertBefore(iframe, where);

  win = iframe.contentWindow || iframe;
  doc = iframe.contentDocument || iframe.contentWindow.document;

  doc.open();
  doc._l = (function (w, d) {
    return function () {
      w.vanishing_global = new Date().getTime();

      var js = d.createElement("script");
      js.src = 'test-vanishing-global.js?' + w.vanishing_global;

      w.name = "foobar";
      d.foobar = "foobar:" + Math.random();
      d.foobar = "barfoo:" + Math.random();
      d.body.appendChild(js);
    };
  })(win, doc);
  doc.write('<body onload="document._l();"></body>');
  doc.close();
})();

The aliasing of win and doc as w and d aren't necessary, it just might make it less confusing because of the misunderstanding of scopes. This way, they are parameters and you have to reference them to access the iframe's stuff. If you want to access the parent's, you still use window and document.

I'm not sure what the implications are of adding methods to a document (doc in this case), but it might make more sense to set the _l method on win. That way, things can be run without a prefix...such as <body onload="_l();"></body>

Why are interface variables static and final by default?

In Java, interface doesn't allow you to declare any instance variables. Using a variable declared in an interface as an instance variable will return a compile time error.

You can declare a constant variable, using static final which is different from an instance variable.

Stop setInterval

we can easily stop the set interval by calling clear interval

var count = 0 , i = 5;
var vary = function intervalFunc() {
  count++;
      console.log(count);
    console.log('hello boy');  
    if (count == 10) {
      clearInterval(this);
    }
}

  setInterval(vary, 1500);

How do I convert a date/time to epoch time (unix time/seconds since 1970) in Perl?

Get Date::Manip from CPAN, then:

use Date::Manip;
$string = '18-Sep-2008 20:09'; # or a wide range of other date formats
$unix_time = UnixDate( ParseDate($string), "%s" );

edit:

Date::Manip is big and slow, but very flexible in parsing, and it's pure perl. Use it if you're in a hurry when you're writing code, and you know you won't be in a hurry when you're running it.

e.g. Use it to parse command line options once on start-up, but don't use it parsing large amounts of data on a busy web server.

See the authors comments.

(Thanks to the author of the first comment below)

How to fully delete a git repository created with init?

If you want to delete all .git folders in a project use the following command:

find . -type f | grep -i "\.git" | xargs rm

This will also delete all the .git folders and .gitignore files from all subfolders

Oracle: Call stored procedure inside the package

To those that are incline to use GUI:

Click Right mouse button on procecdure name then select Test

enter image description here

Then in new window you will see script generated just add the parameters and click on Start Debugger or F9

enter image description here

Hope this saves you some time.

Android Studio suddenly cannot resolve symbols

For me it was a "progaurd" build entry in my build.gradle. I removed the entire build section, then did a re-sync and problem solved.

Finding last index of a string in Oracle

Use -1 as the start position:

INSTR('JD-EQ-0001', '-', -1)

Interface type check with Typescript

I would like to point out that TypeScript does not provide a direct mechanism for dynamically testing whether an object implements a particular interface.

Instead, TypeScript code can use the JavaScript technique of checking whether an appropriate set of members are present on the object. For example:

var obj : any = new Foo();

if (obj.someInterfaceMethod) {
    ...
}

Subtract days from a DateTime

That error usually occurs when you try to subtract an interval from DateTime.MinValue or you want to add something to DateTime.MaxValue (or you try to instantiate a date outside this min-max interval). Are you sure you're not assigning MinValue somewhere?

Removing white space around a saved image in matplotlib

After trying the above answers with no success (and a slew of other stack posts) what finally worked for me was just

plt.gca().set_axis_off()
plt.subplots_adjust(top = 1, bottom = 0, right = 1, left = 0, 
            hspace = 0, wspace = 0)
plt.margins(0,0)
plt.savefig("myfig.pdf")

Importantly this does not include the bbox or padding arguments.

How to globally replace a forward slash in a JavaScript string?

You can create a RegExp object to make it a bit more readable

str.replace(new RegExp('/'), 'foobar');

If you want to replace all of them add the "g" flag

str.replace(new RegExp('/', 'g'), 'foobar');

Add primary key to existing table

Necromancing.
Just in case anybody has as good a schema to work with as me...
Here is how to do it correctly:

In this example, the table name is dbo.T_SYS_Language_Forms, and the column name is LANG_UID

-- First, chech if the table exists...
IF 0 < (
    SELECT COUNT(*) FROM INFORMATION_SCHEMA.TABLES 
    WHERE TABLE_TYPE = 'BASE TABLE'
    AND TABLE_SCHEMA = 'dbo'
    AND TABLE_NAME = 'T_SYS_Language_Forms'
)
BEGIN
    -- Check for NULL values in the primary-key column
    IF 0 = (SELECT COUNT(*) FROM T_SYS_Language_Forms WHERE LANG_UID IS NULL)
    BEGIN
        ALTER TABLE T_SYS_Language_Forms ALTER COLUMN LANG_UID uniqueidentifier NOT NULL 

        -- No, don't drop, FK references might already exist...
        -- Drop PK if exists (it is very possible it does not have the name you think it has...)
        -- ALTER TABLE T_SYS_Language_Forms DROP CONSTRAINT pk_constraint_name 
        --DECLARE @pkDropCommand nvarchar(1000) 
        --SET @pkDropCommand = N'ALTER TABLE T_SYS_Language_Forms DROP CONSTRAINT ' + QUOTENAME((SELECT CONSTRAINT_NAME FROM INFORMATION_SCHEMA.TABLE_CONSTRAINTS 
        --WHERE CONSTRAINT_TYPE = 'PRIMARY KEY' 
        --AND TABLE_SCHEMA = 'dbo' 
        --AND TABLE_NAME = 'T_SYS_Language_Forms' 
        ----AND CONSTRAINT_NAME = 'PK_T_SYS_Language_Forms' 
        --))
        ---- PRINT @pkDropCommand 
        --EXECUTE(@pkDropCommand) 
        -- Instead do
        -- EXEC sp_rename 'dbo.T_SYS_Language_Forms.PK_T_SYS_Language_Forms1234565', 'PK_T_SYS_Language_Forms';

        -- Check if they keys are unique (it is very possible they might not be)        
        IF 1 >= (SELECT TOP 1 COUNT(*) AS cnt FROM T_SYS_Language_Forms GROUP BY LANG_UID ORDER BY cnt DESC)
        BEGIN

            -- If no Primary key for this table
            IF 0 =  
            (
                SELECT COUNT(*) FROM INFORMATION_SCHEMA.TABLE_CONSTRAINTS 
                WHERE CONSTRAINT_TYPE = 'PRIMARY KEY' 
                AND TABLE_SCHEMA = 'dbo' 
                AND TABLE_NAME = 'T_SYS_Language_Forms' 
                -- AND CONSTRAINT_NAME = 'PK_T_SYS_Language_Forms' 
            )
                ALTER TABLE T_SYS_Language_Forms ADD CONSTRAINT PK_T_SYS_Language_Forms PRIMARY KEY CLUSTERED (LANG_UID ASC)
            ;

        END -- End uniqueness check
        ELSE
            PRINT 'FSCK, this column has duplicate keys, and can thus not be changed to primary key...' 
    END -- End NULL check
    ELSE
        PRINT 'FSCK, need to figure out how to update NULL value(s)...' 
END 

Display text on MouseOver for image in html

You can use CSS hover in combination with an image background.

CSS

   .image
{
    background:url(images/back.png);
    height:100px;
    width:100px;
    display: block;
    float:left;
}

.image  a {
    display: none;
}

.image  a:hover {
    display: block;
}

HTML

<div class="image"><a href="#">Text you want on mouseover</a></div>

Javascript - removing undefined fields from an object

var obj = { a: 1, b: undefined, c: 3 }

To remove undefined props in an object we use like this

JSON.parse(JSON.stringify(obj));

Output: {a: 1, c: 3}

How to pass an array into a SQL Server stored procedure

You need to pass it as an XML parameter.

Edit: quick code from my project to give you an idea:

CREATE PROCEDURE [dbo].[GetArrivalsReport]
    @DateTimeFrom AS DATETIME,
    @DateTimeTo AS DATETIME,
    @HostIds AS XML(xsdArrayOfULong)
AS
BEGIN
    DECLARE @hosts TABLE (HostId BIGINT)

    INSERT INTO @hosts
        SELECT arrayOfUlong.HostId.value('.','bigint') data
        FROM @HostIds.nodes('/arrayOfUlong/u') as arrayOfUlong(HostId)

Then you can use the temp table to join with your tables. We defined arrayOfUlong as a built in XML schema to maintain data integrity, but you don't have to do that. I'd recommend using it so here's a quick code for to make sure you always get an XML with longs.

IF NOT EXISTS (SELECT * FROM sys.xml_schema_collections WHERE name = 'xsdArrayOfULong')
BEGIN
    CREATE XML SCHEMA COLLECTION [dbo].[xsdArrayOfULong]
    AS N'<xs:schema xmlns:xs="http://www.w3.org/2001/XMLSchema">
    <xs:element name="arrayOfUlong">
        <xs:complexType>
            <xs:sequence>
                <xs:element maxOccurs="unbounded"
                            name="u"
                            type="xs:unsignedLong" />
            </xs:sequence>
        </xs:complexType>
    </xs:element>
</xs:schema>';
END
GO

How to make Google Fonts work in IE?

After my investigation, I came up to this solution:

//writing the below line into the top of my style.css file
@import url('https://fonts.googleapis.com/css?family=Assistant:200,300,400,600,700,800&subset=hebrew');

MUST OBSERVE: We must need to write the font-weight correctly of this font. For example: font-weight:900; will not work as we have not included 900 like 200,300,400,600,700,800 into the URL address while importing from Google with the above link. We can add or include 900 to the above URL, but that will work only if the above Google Font has this option while embedding.

What does the "assert" keyword do?

Assertions are generally used primarily as a means of checking the program's expected behavior. It should lead to a crash in most cases, since the programmer's assumptions about the state of the program are false. This is where the debugging aspect of assertions come in. They create a checkpoint that we simply can't ignore if we would like to have correct behavior.

In your case it does data validation on the incoming parameters, though it does not prevent clients from misusing the function in the future. Especially if they are not, (and should not) be included in release builds.

Regarding 'main(int argc, char *argv[])'

argc is the number of command line arguments given to the program at runtime, and argv is an array of arrays of characters (rather, an array of C-strings) containing these arguments. If you know you're not going to need the command line arguments, you can declare your main at taking a void argument, instead:

int main(void) {
    /* ... */ 
}

Those are the only two prototypes defined for main as per the standards, but some compilers allow a return type of void as well. More on this on Wikipedia.

Set custom HTML5 required field validation message

This works well for me:

jQuery(document).ready(function($) {
    var intputElements = document.getElementsByTagName("INPUT");
    for (var i = 0; i < intputElements.length; i++) {
        intputElements[i].oninvalid = function (e) {
            e.target.setCustomValidity("");
            if (!e.target.validity.valid) {
                if (e.target.name == "email") {
                    e.target.setCustomValidity("Please enter a valid email address.");
                } else {
                    e.target.setCustomValidity("Please enter a password.");
                }
            }
        }
    }
});

and the form I'm using it with (truncated):

<form id="welcome-popup-form" action="authentication" method="POST">
    <input type="hidden" name="signup" value="1">
    <input type="email" name="email" id="welcome-email" placeholder="Email" required></div>
    <input type="password" name="passwd" id="welcome-passwd" placeholder="Password" required>
    <input type="submit" id="submitSignup" name="signup" value="SUBMIT" />
</form>

enter image description here

Properties private set;

Perhaps, you can have them marked as internal, and in this case only classes in your DAL or BL (assuming they are separate dlls) would be able to set it.

You could also supply a constructor that takes the fields and then only exposes them as properties.

How to redirect to another page using PHP

Although not secure, (no offense or anything), just stick the header function after you set the session variable

 while($row = mysql_fetch_assoc($result))
    {
            $_SESSION["user"] = $username;
    }
header('Location: /profile.php');

How to put a link on a button with bootstrap?

If you don't really need the button element, just move the classes to a regular link:

<div class="btn-group">
    <a href="/save/1" class="btn btn-primary active">
        <i class="glyphicon glyphicon-floppy-disk" aria-hidden="true"></i> Save
    </a>
    <a href="/cancel/1" class="btn btn-default">Cancel</a>
</div>

Conversely, you can also change a button to appear like a link:

<button type="button" class="btn btn-link">Link</button>

Indenting code in Sublime text 2?

The reindent command only works on the currently selected lines unless the "single_line" argument is set to false.

{ "keys": ["f12"], "command": "reindent", "args": {"single_line": false} }

Now, pressing f12 will reindent the entire document.

Saving binary data as file using JavaScript from a browser

This is possible if the browser supports the download property in anchor elements.

var sampleBytes = new Int8Array(4096);

var saveByteArray = (function () {
    var a = document.createElement("a");
    document.body.appendChild(a);
    a.style = "display: none";
    return function (data, name) {
        var blob = new Blob(data, {type: "octet/stream"}),
            url = window.URL.createObjectURL(blob);
        a.href = url;
        a.download = name;
        a.click();
        window.URL.revokeObjectURL(url);
    };
}());

saveByteArray([sampleBytes], 'example.txt');


JSFiddle: http://jsfiddle.net/VB59f/2

QuotaExceededError: Dom exception 22: An attempt was made to add something to storage that exceeded the quota

Here is an expanded solution based on DrewT's answer above that uses cookies if localStorage is not available. It uses Mozilla's docCookies library:

function localStorageGet( pKey ) {
    if( localStorageSupported() ) {
        return localStorage[pKey];
    } else {
        return docCookies.getItem( 'localstorage.'+pKey );
    }
}

function localStorageSet( pKey, pValue ) {
    if( localStorageSupported() ) {
        localStorage[pKey] = pValue;
    } else {
        docCookies.setItem( 'localstorage.'+pKey, pValue );
    }
}

// global to cache value
var gStorageSupported = undefined;
function localStorageSupported() {
    var testKey = 'test', storage = window.sessionStorage;
    if( gStorageSupported === undefined ) {
        try {
            storage.setItem(testKey, '1');
            storage.removeItem(testKey);
            gStorageSupported = true;
        } catch (error) {
            gStorageSupported = false;
        }
    }
    return gStorageSupported;
}

In your source, just use:

localStorageSet( 'foobar', 'yes' );
...
var foo = localStorageGet( 'foobar' );
...

How to include a child object's child object in Entity Framework 5

I ended up doing the following and it works:

return DatabaseContext.Applications
     .Include("Children.ChildRelationshipType");

Getting list of parameter names inside python function

If you also want the values you can use the inspect module

import inspect

def func(a, b, c):
    frame = inspect.currentframe()
    args, _, _, values = inspect.getargvalues(frame)
    print 'function name "%s"' % inspect.getframeinfo(frame)[2]
    for i in args:
        print "    %s = %s" % (i, values[i])
    return [(i, values[i]) for i in args]

>>> func(1, 2, 3)
function name "func"
    a = 1
    b = 2
    c = 3
[('a', 1), ('b', 2), ('c', 3)]

How to use org.apache.commons package?

Download commons-net binary from here. Extract the files and reference the commons-net-x.x.jar file.

No module named 'pymysql'

Sort of already answered this in the comments, but just so this question has an answer, the problem was resolved through running:

sudo apt-get install python3-pymysql

Does delete on a pointer to a subclass call the base class destructor?

No. the pointer will be deleted. You should call the delete on A explicit in the destructor of B.

pycharm running way slow

Every performance problem with PyCharm is unique, a solution that helps to one person will not work for another. The only proper way to fix your specific performance problem is by capturing the CPU profiler snapshot as described in this document and sending it to PyCharm support team, either by submitting a ticket or directly into the issue tracker.

After the CPU snapshot is analyzed, PyCharm team will work on a fix and release a new version which will (hopefully) not be affected by this specific performance problem. The team may also suggest you some configuration change or workaround to remedy the problem based on the analysis of the provided data.

All the other "solutions" (like enabling Power Save mode and changing the highlighting level) will just hide the real problems that should be fixed.

How to convert Java String into byte[]?

Simply:

String abc="abcdefghight";

byte[] b = abc.getBytes();

Reading a cell value in Excel vba and write in another Cell

surely you can do this with worksheet formulas, avoiding VBA entirely:

so for this value in say, column AV S:1 P:0 K:1 Q:1

you put this formula in column BC:

=MID(AV:AV,FIND("S",AV:AV)+2,1)

then these formulas in columns BD, BE...

=MID(AV:AV,FIND("P",AV:AV)+2,1)
=MID(AV:AV,FIND("K",AV:AV)+2,1)
=MID(AV:AV,FIND("Q",AV:AV)+2,1)

so these formulas look for the values S:1, P:1 etc in column AV. If the FIND function returns an error, then 0 is returned by the formula, else 1 (like an IF, THEN, ELSE

Then you would just copy down the formulas for all the rows in column AV.

HTH Philip

How to set java_home on Windows 7?

Find JDK Installation Directory

First you need to know the installation path for the Java Development Kit.

Open the default installation path for the JDK:

C:\Program Files\Java

There should be a subdirectory like:

C:\Program Files\Java\jdk1.8.0_172

Note: one has only to put the path to the jdk without /bin in the end (as suggested on a lot of places). e.g. C:\Java\jdk1.8.0_172 and NOT C:\Java\jdk1.8.0_172\bin !


Set the JAVA_HOME Variable

Once you have the JDK installation path:

  1. Right-click the My Computer icon on your desktop and select Properties.
  2. Click the Advanced tab, then click the Environment Variables button.
  3. Under System Variables, click New.
  4. Enter the variable name as JAVA_HOME.
  5. Enter the variable value as the installation path for the Java Development Kit.
  6. Click OK.
  7. Click Apply Changes.

Note: You might need to restart Windows

The complete article is here, on my blog: Setting JAVA_HOME Variable in Windows.

How to put text in the upper right, or lower right corner of a "box" using css

You need to put "here" into a <div> or <span> with style="float: right".

C++, how to declare a struct in a header file

You can't.

In order to "use" the struct, i.e. to be able to declare objects of that type and to access its internals you need the full definition of the struct. So, it you want to do any of that (and you do, judging by your error messages), you have to place the full definition of the struct type into the header file.

How can I keep Bootstrap popovers alive while being hovered?

Here is a solution I devised that seems to work well while also allowing you to use the normal Bootstrap implementation for turning on all popovers.

Original fiddle: https://jsfiddle.net/eXpressive/hfear592/

Ported to this question:

<a href="#" id="example" class="btn btn-danger" rel="popover" >hover for popover</a>

$('#example').popover({
    html : true,
    trigger : 'hover',
    content : function() {
        return '<div class="box"></div>';
    }
}).on('hide.bs.popover', function () {
    if ($(".popover:hover").length) {
      return false;
    }                
}); 

$('body').on('mouseleave', '.popover', function(){
    $('.popover').popover('hide');
});

How to square all the values in a vector in R?

How about sapply (not really necessary for this simple case):

newData<- sapply(data, function(x) x^2)

Summernote image upload

UPLOAD IMAGES WITH PROGRESS BAR

Thought I'd extend upon user3451783's answer and provide one with an HTML5 progress bar. I found that it was very annoying uploading photos without knowing if anything was happening at all.

HTML

<progress></progress>

<div id="summernote"></div>

JS

// initialise editor

$('#summernote').summernote({
        onImageUpload: function(files, editor, welEditable) {
            sendFile(files[0], editor, welEditable);
        }
});

// send the file

function sendFile(file, editor, welEditable) {
        data = new FormData();
        data.append("file", file);
        $.ajax({
            data: data,
            type: 'POST',
            xhr: function() {
                var myXhr = $.ajaxSettings.xhr();
                if (myXhr.upload) myXhr.upload.addEventListener('progress',progressHandlingFunction, false);
                return myXhr;
            },
            url: root + '/assets/scripts/php/app/uploadEditorImages.php',
            cache: false,
            contentType: false,
            processData: false,
            success: function(url) {
                editor.insertImage(welEditable, url);
            }
        });
}

// update progress bar

function progressHandlingFunction(e){
    if(e.lengthComputable){
        $('progress').attr({value:e.loaded, max:e.total});
        // reset progress on complete
        if (e.loaded == e.total) {
            $('progress').attr('value','0.0');
        }
    }
}

Changing text color of menu item in navigation drawer

try this in java class

yourNavigationView.setItemTextColor(new ColorStateList(
            new int [] [] {
                    new int [] {android.R.attr.state_pressed},
                    new int [] {android.R.attr.state_focused},
                    new int [] {}
            },
            new int [] {
                    Color.rgb (255, 128, 192),
                    Color.rgb (100, 200, 192),
                    Color.WHITE
            }
    ));

string.Replace in AngularJs

The easiest way is:

var oldstr="Angular isn't easy";
var newstr=oldstr.toString().replace("isn't","is");

Git Clone - Repository not found

I was also having same issue. I was trying to clone the repo which was private and my git installed in osx has keychain which was not allowing me to clone the repo...

I tried

git clone https://username:[email protected]/NAME/repo.git 

but it didn't work as my password was containing the field @.

I just ran

git credential-osxkeychain erase
host=github.com
protocol=https

command and press enter and it worked perfectly fine. Actually you need to remove the keychain already stored in the osx.

The request failed or the service did not respond in a timely fashion?

If you recently changed the password associated with the service account:

  1. Start SQL Server Configuration Manager.
  2. Select SQL Server Services in the left pane.
  3. Right click the service you are trying to start in the right pane and click Properties.
  4. Enter the new Password and Confirm password.

bash: shortest way to get n-th column of output

Note, that file path does not have to be in second column of svn st output. For example if you modify file, and modify it's property, it will be 3rd column.

See possible output examples in:

svn help st

Example output:

 M     wc/bar.c
A  +   wc/qax.c

I suggest to cut first 8 characters by:

svn st | cut -c8- | while read FILE; do echo whatever with "$FILE"; done

If you want to be 100% sure, and deal with fancy filenames with white space at the end for example, you need to parse xml output:

svn st --xml | grep -o 'path=".*"' | sed 's/^path="//; s/"$//'

Of course you may want to use some real XML parser instead of grep/sed.

Display Image On Text Link Hover CSS Only

I did something like that:

HTML:

<p class='parent'>text text text</p>
<img class='child' src='idk.png'>

CSS:

.child {
    visibility: hidden;
}

.parent:hover .child {
    visibility: visible;
}